Download More FREE Past Questions at www.prep.ng UNILORIN POST-UTME PAST QUESTIONS 2006-2018

Instruction: For question 7-8, choose the UNILORIN 2006 POST-UTME option that is nearest in meaning to the word QUESTIONS underlined. 1. Complete the following series: 50, 40, 100, 90, 150, __? 7. I wonder what will be left of his essay when A. 200 the extraneous material is deleted. B. 180 A. Superfluous C. 140 B. erroneous D. 300 C. relevant D. main 2. A class of 27 students has a 33% pass. How many of them passed? 8. His success may be described as a pyrrhic A. 9 victory. B. 10 A. costly C. 12 B. a deserving victory D. 21 C. an easy victory D. indecisive 3. A graduating class has 420 candidates. If 5% of them are in honours roll and the 9.The constituency of the senator is ___ the requirement for being in honours roll is to federal representative. score the minimum of 75%, how many A. larger than candidates scored below 75%? B. smaller than A. 400 C. equal to B. 399 D. twice that of C. 200 D. 199 10. Which of the following statements about language is incorrect? 4. A graduating class has 420 candidates. If A. language barrier often created problems 5% of them are in honours roll and the B. language barrier often led to discrimination requirement for being in honours roll is to C. language barrier often bred suspicion score the minimum of 75%, how many D. language barrier often created friendship candidates made the honours roll? A. 321 11.Computer logic is based on two digits B. 40 namely: C. 30 A. 0,1 D. 21 B. 1,10 C. 1,2 5. If the class average of a 100-student class D. 0,9 is 50% and 25%of the students scored 70%. How many scored below 12.The numbers 0 and 1 in computer language 75%? is referred to as_____. A. 50 www.prep.ngA. byte B. 75 B. base C. 60 C. bit D. 25 D. term

6. The take-over progression to the presidency 13. INEC stands for ___. in case of death or step-aside of the president A. Independent National Electoral Commission is ____. B. Independent Nigerian Elections Commission A. vice president, senate president, and C. Independent National Electric Commission speaker of the house of representative D. Internal National Electoral Commission B. vice president, speaker of the house, and senate president 14. The solar eclipse occurs when ____. C. senate president, speaker of the house, and A. the sun comes in between the moon and the vice president earth D. speaker of the house, vice president, and B. the earth comes in between the sun and the senate president. moon 1

Download More FREE Past Questions at www.prep.ng

C. the moon comes in between the sun and the earth 23. If 6 men can do a piece of work in 2 days, D. None of the above how long will it 2 men? A. 4days 15. The removal of top soil by water or wind is B. 6days called ____ C. 5days A. Soil wash D. 12days B. Soil erosion C. Soil creep 24. Internet connectivity is based on ___. D. Silting of soil A. radio communication B. computer communication 16. Who invented the telephone? C. television communication A. Thomas Alva Edison D. video communication. B. Galileo C. Alexander Graham Bell 25. There was lightning and this was followed D. Thomas Graham Bell by thunder. The boy who saw the lightning flash was not afraid of lightning but got scared 17. Albert Einstein was a famous ____. when the thunder roared. Which of these is A. Physician dangerous? B. Chemist A. lightning C. Physicist B. thunder D. Biologist C. both D. none 18. If 6 men can dig a well in 3 days, how many men will dig it in 2 days at the same 26. A man was travelling from Lagos to Ibadan rate? in a bus and on the opposite side of the road a A. 9men woman was travelling to Lagos from Ibadan in B. 12men another bus. Suddenly a car passed the bus C. 16 men the man was travelling in. How does the man D. 15 men feel with respect to the car? A. bus faster than the car 19. A trader bought a good for ₦20 and sold it B. bus moving backwards for ₦20. What was the gain percent? C. the driver of the car is slowing down A. 100% D. the bus driver was no longer moving at his B. 10% speed before the car approached them C. 1% D. 0% 27. Despite all preparation, the wedding did not____. 20. Some goods were sold for #11,000 at a A. come along profit of 10% of the cost price. What did the B. come off goods cost? C. come on A. ₦10,000 D. come up B. ₦11,100 www.prep.ng 2 C. ₦1,200 28. A man owns /3 of the market research 3 D. ₦13,200 business and sells /4 of his shares for N75,000. What is the value of business? 21. How many 40K can we get in ₦16,000? A. N150,000 A. 400 B. N13,000 B. 300 C. N240,000 C. 200 D. N34,000 D. 100 29. Which of the following numbers should be 22. Express two Million, two hundred thousand added to 11158 to make it exactly divisible by and 50 in figures. 77? A. 2200050 A. 9 B. 2020050 B. 8 C. 2205000 C. 7 D. 2020050 D. 5 2

Download More FREE Past Questions at www.prep.ng

C. 9899 30. On selling 3 articles at the cost of 4 D. 9801 articles, there will be profit of ___. 36.1 A. 25% 38. Find the root value of /102.4? 100 61 B. /3% A. /34 75 19 C. /2% B. /31 19 D. 40% C. /32 19 D. /33 31. By selling an article for ₦40, there is a loss of 40%, by selling it for ₦80, there would be 39. A shop keeper sold a T.V. set for ₦17,940 A. Gain of 20% with a discount of 8% and earned a profit of B. Loss of 10% 19.6%, what would have been the percentage C. Loss of 20% of profit earned if no discount was offered? D. Gain of 10% A. 24.8% B. 25% 32. A number consists of 2 digits whose sum is C. 26.4% 8, if 8 is subtracted from the number, the D. None of the above digits interchange their place. The number is ____. 40. If (2x-y) = 4, then (6x-3y) =? A. 44 A. 15 B. 35 B. 12 C. 62 C. 18 D. 33 D. 10

33.The sides of a triangle are in the ratio of 41.A clock is set at 8am. The clock gains 3:5:7 and its perimeter is 30cm. The length of 10min in 24hrs. What will be the true time the greatest side of the triangle in cm is ____. when the clock indicates 1pm on the following A. 6 day? B. 10 A. 48 minutes past 12 C. 14 B. 38 minutes past 12 D. 16 C. 28 minutes past 12 D. 25 minutes past 12 34. The radius of a right circular cone is 30cm and its height is 4cm, the curved surface of the 42. A person has 4 coins each of different cone will be ____. denomination, what is the number of different A. 12sq.cm sums of money the person can form (using B. 15sq.cm one or more coins at a time.? C. 18sq.cm A. 16 D. 21sq.cm B. 15 C. 12 7 16 21 35. The fractions /11; /20; /22 when D. 11 arranged in descending order is 7 16 21 A. /11; /20; /22 www.prep.ng43. Which number is the odd one out of 9678, 21 7 16 B. /22; /11; /20 4572, 5261, 3527, 7768? 21 16 7 C. /22; /20; /11 A. 9678 7 21 16 D. /11; /22; /20 B. 4572 C. 5261 36. If 10% of a number is subtracted from it, D. 3527 result is 1800. The number is ___. A. 1900 44. A radio when sold at a certain price gives a B. 2000 gain of 20%, what will be the gain percent if C. 2100 sold for thrice the price? D. 2140 A. 280 B. 270 37.The number proceeding 9909 which is a C. 290 perfect square is ___. D. 260 A. 9908 B. 9900 3

Download More FREE Past Questions at www.prep.ng

45.A pencil 20cm long was recorded as 22cm D. Kid long. What is the percentage error? A. 22% 54.The full meaning of GSM is _____. B. 20% A. Got Sent Messages C. 10% B. Global Service Mobile D. 5% C. General System manager D. Global System for Mobile communications 46. A student made a 5% error in the measurement of a 50cm long stick. What was 55. Some green are blue; no blue is white. the student’s measurement? A. Some green is white A. 50cm B. No white is green B. 60cm C. No green is white C. 512.5cm D. None of the above D. 55cm 56.I have trouble _____. 47. Complete the sentence. As soon as he A. to remember my password entered the room, he ____ off his shirt. B. to remembering my password A. Take C. remember my password B. Takes D. remembering my password C. Took D. Taken 57. We judge people by what they do rather than what they say. Why? 48. Ngozi is the ___of the three sisters. A. Words are sometimes hard to understand A. Taller B. Words often have more than one meaning B. Tallest C. What people do, tells us what they are C. Most tall really like D. More taller D. What people do, is not always the right thing 49. For a paper kite to fly high, it needs ______58.Why do we breast-feed? A. fire A. Because it is the cheapest way of feeding B. fuel babies. C. air B. Because we have plenty of milk. D. water C. Because it is the easiest food to prepare. D. Because no other food is so nourishing to 50. Compatriots in the first stanza of the babies. National Anthem refers to………. A. soldiers 59. Meat is cooked before it is eaten because: B. Nigerians A. It tastes better that way. C. companies B. It smells better when cooked. D. foreigners C. Housewives like cooking meat. D. Cooking kills germs. 51. Braille is used by ------www.prep.ng A. illiterate people 60. Which tells you best what plenty is? B. blind people A. A great deal. C. short sighted people B. Enough. D. old people C. More than enough. D. All that you want. 52.The young of a goat is called A. Son 61.The words telegraph, telephone, television, B. Kid all contains- “tele” what does it mean? C. calf A. It tells us that these three things are D. Baby worked by electricity. B. It tells us that these things are scientific. 53. The young of a cow is called ______. C. It tells us that these things enable one to A. Ewe deal with things at a distance. B. Kitten D. It simply means “tell or say”. C. Calf 4

Download More FREE Past Questions at www.prep.ng

2 62.If bread in code is csfbe, then cbe 70. After spending /3 of her money on food, a means...... woman was left with ₦250. How much money A. Bed did she have originally? B. Ear A. ₦250 C. Bad B. ₦500 D. Red C. ₦750 D. ₦1,000 63. If scraping in code is rbqzohmf, then fzor means...... 71. A man with N5,000 spent 1/5th at the A. Grin Chemist and ¼ at the Electric shop. How much B. gaps money did he have left? C. gasps A. ₦2,750 D. gain B. ₦3,750 C. ₦4,000 64.If ynncyjgle in code is appealing, then nyle D. ₦1,000 means...... 1 A. Pane 72. A girl spent /2 of her money at the market 1 B. Pale and /5th of the remaining amount at the C. Pang bakery. If she had ₦1,000 originally, how D. Peal much has she left? A. ₦100 65.If amlypqcjw in code is coarsely, then alpq B. ₦200 means...... C. ₦300 A. Case D. ₦400 B. Coal C. Clay 73. If Ayuba bought 5 oranges for ₦7, how D. Core many oranges of the same type can he buy for ₦49? 66. If ftgcokpi in code means dreaming, then A. 15 oranges ogcp means...... B. 35 oranges A. Maid C. 25 oranges B. Made D. 30 oranges C. Mean D. Mend 74. If a can of malt drink costs ₦65, how much is a dozen cans of malt drink? 67. If the exchange rate is US$ 1 = ₦150, how A. ₦780 much is US$ 500 in naira? B. ₦715 A. ₦150,000 C. ₦650 B. ₦500,000 D. ₦585 C. ₦7,500 D. ₦75,000 75.Bunmi can hoe a yam plot in 5hrs. and Dele can hoe it in 4hrs, If the two men hoe, what 68. How much is ₦3,000www.prep.ng worth in US$ if the fraction of the yam plot will be left to hoe after exchange rate is US$ 1 = ₦120? 2hrs? A. US$ 65 A. ¼ 1 B. US$ 25 B. /5 1 C. US$ 45 C. /8 1 D. US$ 35 D. /10

69. Which is the greatest of the following 76. What fraction of the yam plot will they hoe fractions? in 1hr? 7 A. /9 A. ¼ 9 B. ¾ B. /20 5 1 C. /8 C. /9 2 9 D. /3 D. /10

77. In an election the winner received 3/7 of the total vote: If the total number of votes 5

Download More FREE Past Questions at www.prep.ng cast were 700000, what number of votes did 79. B 80. A the winner receive? A. 100,000 votes B. 200,000 votes C. 300,000 votes D. 400,000 votes

78. If the total votes cast were 1.4 million, what total votes did the losers receive? A. 800,000 votes B. 600,000 votes C. 1 million votes D. 1.2 million votes

79. If there were 3 candidates at the election 5 and one of the losers received /14 of the votes, what number of votes did the other loser receive? A. 100,000 votes B. 150,000 votes C. 200,000 votes D. 250,000 votes

80. A factory increased its annual production of radios from 40,000 to 50,000. Calculate the percentage increase in production. A. 25% B. 10% C. 15% D. 20%

ANSWERS TO UNILORIN 2006 POST UTME SOLUTION

1. C 2. A 3. B 4. D 5. B 6. A 7. A 8. A

9. A 10. D 11. A 12. C 13. A 14. C 15. B

16. C 17. C 18. A 19. D 20. A 21. A 22. A

23. B 24. B 25. A 26. B 27. B 28. A 29. C

www.prep.ng

30. B 31. A 32. D 33. C 34. B 35. C 36. B

37. D 38. C 39. D 40. B 41. A 42. B 43. D

44. D 45. C 46. C 47. C 48. B 49. C 50. B

51. B 52. B 53. C 54. D 55. A 56. D 57. C

58. D 59. D 60. A 61. C 62. C 63. B 64. C

65. A 66. C 67. D 68. B 69. A 70. C 71. A

72. D 73. B 74. A 75. D 76. B 77. C 78. A

6

Download More FREE Past Questions at www.prep.ng UNILORIN POST UTME PAST QUESTIONS 2007/2008

B. Mr. Taiwo Akinyemi 1.The first female to drive a car in Nigeria is C. Mr. Taiye Akinkumi _____. D. None of the above A. Mrs. Olufunmilayo Ransome Kuti B. Mrs. Bolanle Awe 10. University of Ilorin is a _____ generation C. Mrs. Mary Herbert University. D. None of the above A. 1st B. 2nd 2. Who was the first prime minister of Nigeria? C. 3rd A. Sultan of Sokoto D. 4th B. Alh.Tafawa Balogun C. Alh. Tafawa Balewa 11. Which of these is the first Nigerian D. Alh. Tafa Balewa University? A. Obafemi Awolowo University 3. The largest bird in the world is _____. B. University of Ilorin A. Vulture C. University of Ibadan B. Eagle D. C. Ostrich D. Swan 12. What is the meaning of CBT? A. Computer Basic Technology 4. The full meaning of the acronym “GSM” is B. Computer Basic Test ____. C. Computer Based Test A. Global System Mobile D. All of the above B. General System Mobile Communication C. Global system of Mobile communication 13. Nigeria Federal Capital Territory shifted D. General System Mobile from Lagos to Abuja in what year? A. 1991 5. Nigeria as a country is not a member of B. 1989 which of the following: C. 1990 A. U.N.O D. 1992 B. ECOWAS C. AU 14. Nigeria celebrated her Silver Jubilee in the D. None of the above year …………… A. 1985 6. Nigeria changed from Pounds to Naira in B. 2003 ____. C. 1995 A. 1972 D. 2008 B. 1971 C. 1970 15. People who carry out experiments to D. 1975 investigate nature are called ____. A. Scientist 7. Whose photograph appearswww.prep.ng in the ₦1000 B. Experimentalist note? C. Experimentist A. Clement Isong and Alh Tafawa Alewa D. Scientia B. Clement Isong and Alh Aliyu Kingibe C. Clement Isong and Aliyu Maibornu 16. Objects thrown up, always come down due D. Alh Maikingibe and Clement Effiong to ______. A. kinetic force 8. The Green and White colours in Nigeria Flag B. potential force symbolize____. C. gravitational force A. Agriculture and Finance D. Gravitational pull B. Peace and Prosperity C. Agriculture and Transparency 17. Human beings have different types of teeth D. Agriculture and peace and hence are referred to as ____. A. Homodont 9. Who designed the National flag? B. Homodent A. Mr. Taiwo Akinkunmi C. Heterodent 7

Download More FREE Past Questions at www.prep.ng

D. Isodent A. ₦56.50 B. ₦57.50 18. ICT Means ____. C. ₦57.00 A. Information and Communication Technology D. ₦59.50 B. Information Communication Technology C. International Communication Technology 25. Dapo was knock down by a car with plate D. Internal Community Technology number HA539KST. The police on duty picked the car plate number in a reverse order 19. Which of the following is proteinous in as...... nature? A. HA 359KST A. groundnut B. KST 593HA B. millet C. HA 539KTS C. beans D. TSK935AH D. guinea corn 26. A rancher plans to add a post between 20. A man having 20 litres of petrol in his car, every two posts of an open-ended (straight) uses it for 600 km. How many kilometres will fence. If he currently has 10 posts in the the same man travel with 19.25 litres. fence, how many does he need to insert? A. 577.5km A. 5 B. 576.2 km B. 9 C. 587.5km C. 10 D. 586.2km D. 20

21.Which side of the chest is the heart 27.Express 398753 correct to three significant normally located? figures A. left A. 398000 B. right B. 398700 C. centre C. 398800 D. part D. 399000

22.. Ikeja is to Lagos as ------is to Akwa- 28. A boy estimated his transport fare for a Ibom. journey as ₦190 instead of ₦200. Find the A. Calabar percentage error in his estimate B. Mecca A. 5% C. Uyo B. 47.5% D. Ibadan C. 5.26% D. 95% 23. Subtract ninety-nine from nine hundred and ninety and then, add it to eight hundred 29. In a bag of oranges, the ratio of good and sixty-five. What will be the final answer? ones to bad ones is 5:4. If the number of bad A. 1756 oranges in the bag is 36, how many oranges B. 1855 C. 1954 www.prep.ngare there altogether? D. 1180 A. 72 B. 81 24. If John starts work at 8.845am and C. 54 finishes at 5.15pm. He has 90 minutes of D. 45 break. How many hours does he work in 5 Use the following Instructions to days? A.38 answer questions 30-41. Choose a word B. 39 which has the same or nearly the same C. 35 meaning as the first word.

D. 40 30. Feeble:

A. Strong 25. A restaurant bill is made up of the B. Weak following: ₦12.50 for starters, ₦28.55 for main C. Tired courses and ₦8.95 for deserts, plus a ₦17.50 D. Little service charge. How much is the bill? 8

Download More FREE Past Questions at www.prep.ng

31. Astonish: 40.Prohibit: A. Asunder A. Advertise B. Attack B. Allow C. Surprise C. Forbid D. Sensible D. Destroy

32. Miserable: 41. Excitement: A. Mischief A. Enthusiasm B. Wretched B. Zeal C. Mistake C. Fuss D. Wicked D. Success

33. Stubborn: SIMILES A. Obstinate B. Sturdy Complete questions 42- 48. C. Fearful D. Strong 42. Her gown is as green as ____. A. grace Choose a word which has the same or B. grass nearly the same meaning as the first C. flag word. D. butter

34. Permanent: 43. Olu is as hungry as a______. A. Temporary A. orphan B. Old B. wolf C. Lasting C. termites D. Complete D. thug

35. Boss: 44. He is as mischievous as _____. A. Governor A. Satan B. Servant B. monkey C. Mistress C. gazelle D. master D. tortoise

36. Dear: 45. He is as industrious as _____. A. precious A. an ant B. cheap B. elephant C. nice C. worker D. useless D. messenger

37. Elegance: 46. As merciless as a _____. A. Grace www.prep.ngA. grave B. Display B. grasshopper C. Safety C. catastrophe D. Ugly D. catapult

38.Thankfulness: 47. As brittle as _____. A. greatness A. pure water B. gratitude B. glass C. meanness C. bintu D. goodness D. wood 48. As beautiful as ______. 39. Study: A. rainbow A. Learn B. brass B. Enjoy C. peacock C. Teach D. lamb D. Remember 9

Download More FREE Past Questions at www.prep.ng

49. Because of his ...... nature, Ayo was B. Vociferous tricked out of his watch C. Stuttering A. Kind D. Gesticulating B. Pliable C. Uncompromising 58. Making a phone call instead of paying a D. Uncalculating visit represent a ...... trend A. Sudden new major social 50. He says he ...... find me a job, but will B. New social sudden major accommodate me. C. New sudden major social A. Could not D. Sudden social new major B. Was not able to C. Is not able to 59. She was the first woman to D. Cannot be able to stand...... election to parliament. A. to 51. The idle steward could not see ...... on B. for the wall, and he lost his job. C. through A. The writing D. by B. What is written C. What will be written 60. They waited in a state of feverish anxiety D. The handwriting ...... their mother to come home. A. till 52. Yesterday in the hall, Ola said that...... B. to his watch C. until A. he had mislaid D. for B. he had forgot C. he has misplaced 61 Complete the series English, French, Hindi, D. he have lost Arabic A. Swahili 53. Government should invest more in...... B. Language training C. Dialect A. Teacher’ D. Africa B. Teacher C. Teachers 62. If you were to add all odd numbers D. Teachers’ between 1 and 11 both inclusive, the result would be ___ 54. The team has benefitted from the coach's A. Even ...... of experience. B. Odd A. Minefield C. Impossible to say B. Reservoir D. None of the above C. Wealth D. World 63.When Jack, James, Jim and Jane stand by age, Jack being the youngest stands first while 55. There was ...... anywww.prep.ng traffic as he drove James brings up the rear. However, when they home. stand by height, Jim being the shortest stands A. Hardly first while James comes to the third spot. In B. Normally both lines Jane remains at the second position. C. Usually Who is immediately younger than James? D. No A. Jack B. James 56. I look forward to ...... you next week. C. Jim A. Seen D. Jane B. Seeing C. Be seeing 64.The day after tomorrow is three days before D. See a Monday. What day is it today? A. Monday 57. The girl cannot speak with a clear voice, B. Tuesday she is always ...... C. Wednesday A. Blabbing D. Thursday 10

Download More FREE Past Questions at www.prep.ng

73. A man’s annual salary is N132,000. What 65.Which is the next number is the series? 18, is his monthly salary? 13, 9, 6, 4, ___ A. ₦11,000 A. 1 B. ₦12,000 B. 2 C. ₦13,000 C. 3 D. ₦22,000 D. 4 74. A man earns ₦20,000 a month. What does 66. Steam is to water as liquid is to _____. he earn in a year? A. Ice A. ₦200,000 B. Solid B. ₦240,000 C. Vapour C. ₦220,000 D. Snow D. ₦280,000

67.Sir Richard’s constitution was introduced in 75. In April, Mrs. Ngozi bought a bag of corn Nigeria in _____. for ₦1,200. When she bought more in June, A. 1946 the price had gone up by 20%. How much did B. 1950 she pay in June for a bag of corn? C.1964 A. ₦1,300 D. 1864 B. ₦1,400 C. ₦N1,440 68.The Central Bank of Nigeria was established D. ₦1,340 in _____. A. 1920 76. A trader made a loss of 10% on a B. 1908 motorcycle he sold for ₦54,000: How much C. 1959 should he have sold the motorcycle to avoid D. 1950 any loss? A. ₦56,000 69.Headquarters of NYSC is at B. ₦58,000 A. Abuja C. ₦60,000 B. Aba D. ₦62,000 C. Kano D. Lagos 77. How much should he have sold the motorcycle to make a gain of 10%? 70. The Nigeria Civil War started on ------. A. ₦60,000 A. 3rd October, 1970 B. ₦62,000 th C. ₦64,000 B. 6 July 1967 D. ₦66,000 C. 16th July 1967 D. 1st April 1976 78. A train moves at a speed of 55km/hr: How long will it take to cover 385km? 71. A can of Cola was labelled as containing A. 5hrs. 330ml of fluid but was fowww.prep.ngund to contain 341ml. B. 7hrs. What was the percentage error? C. 9hrs. A.33.0% D. 11hrs. B. 34.1% C. 3.33% 79. How far will it go in 3hrs? D. 3.41% A. 165km B. 175km 72. A surveyor measured a road as being C. 185km 95km with 5% underestimation. What is the D. 195km true length of the road? A. 90km 80. Who hosted the 2006 world cup? B. 95km A. Germany C. 100km B. Greece D. 105km C. France D. England

11

Download More FREE Past Questions at www.prep.ng

ANSWERS TO 2007 POST-UTME QUESTIONS

1. A 2. C 3. C 4. C 5. D 6. A 7. C 8. D

9. A 10. B 11. C 12. C 13. A 14. A 15. A

16. D 17. C 18. A 19. A 20. A 21. A 22. C

23. A 24. C 25. B 26. D 27. D 28. A 29. B

30. B 31. C 32. B 33. A 34. C 35. D 36. A

37. A 38. B 39. A 40. C 41. A 42. B 43. B

44. B 45. A 46. A 47. B 48. A 49. B 50. C

51. D 52. A 53. B 54. C 55. A 56. B 57. C

58. A 59. B 60. D 61. A 62. A 63. C 64. C

65. C 66. B 67. A 68. C 69. A 70. B 71. C

72. C 73. A 74. B 75. C 76. C 77. D 78. B

79. A 80. A

www.prep.ng

12

Download More FREE Past Questions at www.prep.ng UNILORIN POST UTME PAST QUESTIONS 2008/2009

1. Which of these is active in the night? D. Bat A. bird E. Pigeon B. squirrel C. bat 10.Which of the following is an amphibian? D. lizard A. Whale B. Toad 2.Export goods are………… C. Lizard A. Goods and services produced in the country D. Pigeon B. Manufacture outside the country E Tilapia C. Goods produced for sale in the country D. Goods produced for sale outside the country 11.Which of the following is a reptile? A. Whale 3. The sun is the major source of ------and ---- B. Toad A. power and heat C. Lizard B. light and heat D Pigeon C. strength and heat E Tilapia D. light and darkness 12. President Jonathan’s speech ______at 9pm 4. A female dog is called a/an …………. yesterday. A. kitten A. had being broadcast B. bitch B. had been broadcast C. ewe C. was broadcasted D damp D. was broadcast

5.The Armed Forces of Nigeria comprise of ……. 13.The way to stop frivolous publications is to A. the Army, Custom, police ____ the press B. Navy, Immigration, Air-force A. bribe C. The police, Air-force, Navy B. gag D. Army, Air-force, Navy C. jail D. shackle 6. A man who mends shoes is known as _____. 14. Which of the following is correct? A. leather maker A. Acomodation B. shoe shiner B. Accomodation C. cobbler C. Accommodation D. artist D. Acommodation E. Acomoddation 7. At what time of the day is the sun at the zenith? 15. A man is four times as old as his son. The A. 8am difference between their ages is 36. Find the B. 10am sum of their ages C. 4pm www.prep.ngA. 45years D. 12 Noon B. 48years E. 2pm C. 60years D. 74years 8. Supply of electricity in Nigeria is the responsibility of _____. 16. A student spends ¼ of his pocket money 1 A. National Electric Power Authority on books and /3 on a shirt. What fraction B. Power Holding Company of Nigeria remains? 5 C. National Atomic Power Agency A. /6 7 D. National Energy Commission B. /12 1 E. National Emergency Management Agency C. /6 5 D. /12 9. Which of these is not a bird? A. Kite 17. The _____ in today’s issue of The Punch B. Eagle Newspaper focused on inflation C. Vulture A. title 13

Download More FREE Past Questions at www.prep.ng

B. editorial A. 13 C. headline B. 11 D. story C. 12 D. 14 18.How many sides in a pentagon? A. 8 26.Two-third of a class passed English B. 6 Language as a subject. The number of C. 7 students who failed English Language will be? D. 5 1 A. /3 E. 9 1 B. /2 1 19. If a typist can type at the rate of 50 C. /4 words/minute, how long will it take to type a D. 3/ 7500-word document? 4 A. 3Hrs B. 4Hrs 27.“My Heart Jumped into my mouth” What is 1 the meaning of the above statement? C. 2 /2Hrs A. To be confused D. 1Hr B. To be hungry C. To be afraid 20.What is your percentage profit margin if D. To be discouraged you bought an item for N600 and sold it for N750? Chose the lettered word or phrase that is A. 600% most nearly opposite in meaning to the B. 750% words in capital letters. Use it to answer C. 150% question 28-29 D. 25% E. 100% 28. EXEMPLARY A. Foreign Choose a word opposite in meaning to the B. Imitate first word. Use it to answer questions 21- C. Additional 24 D. Outstanding

21. Liberty: 29. ABUNDANT A. Freedom A. Surplus B. Captivity B. Few C. Stadium C. Brilliance D. Guilty D. Birth

22.Dissent: 30.A promotional discount of 15% is offered on A. Dissent a new coat which previously cost ₦18,000. B. Undissent What is the discounted price of the coat? C. Assent A. ₦15,300 D. Indissent www.prep.ng B. ₦14,000 C. ₦15,500 23. Sharp: D. ₦16,000 A. Clever B. Blunt Choose a word opposite in meaning to the C. Keen first word. Use it to answer question 31- D. Even 37.

24. Conductor: 31. Above: A. Driver A. over B. Conductress B. below C. Passenger C. high D. Wire D. low

25. How many Lunar months make one year 32. Liberty: 14

Download More FREE Past Questions at www.prep.ng

A. freedom B. Madagascar B. captivity C. Cape Verde C. stadium D. Zanzibar D. guilty 42. In which state is Lake Chad situated? 33. Dissent: A. Adamawa A. dissent B. Borno B. Undissent C. Taraba C. Assent D. Sokoto D Indissent 43. The black race is called ____. 34. Sharp: A. Caucasian A. clever B. Red Indian B. blunt C. Mongolian C. Keen D. Negroid D. even 44.The headquarters of United Nations is 35. Conductor: located at ____. A. driver A. Geneva B. conductress B. New York C. Passenger C. Rome D. Wire D. Washington

36. Spendthrift: 45. Seun‘s friend was knock down by a car, he A. miser tried to copy the car plate number HA539KST. B. loan Which one of the following is a correct copy of C. Foolish the car plate number? D. happy A. HA 359KST B. HA 593KST 37. Profane: C. HA 539KTS A. beautiful D. HA 539KST B. sacred C. Profuse 46. Danladi is to transfer some money into the D. Stiff account of his friend which reads 040000111906. Which one of the following is a 38.Which of the following rivers is not found in correct copy of the account number of his Nigeria? friend? A. River Kaduna A. 040000111960 B. Cross River B. 040000111906 C. River Niger C. 04000111906 D. River Nile D. 04000011906 E. River Benue www.prep.ng47. Bible is to a priest, as ____to a don. A. Qur’an 39.The longest river in Nigeria is river ___. B. Books A. Benue C. Music B Hadejia D. Drum C. Niger D. Osun 48. ____ is the Head of British Commonwealth of Nations. 40. Nigeria is bounded in the south by ____. A. President Umaru Musa Yar'Adua A. Togo B. David Cameron B. Atlantic Ocean C. Queen Elizabeth II C. Pacific Ocean D. Barack Obama. D. Cameroon 49. In which geo-political zone of Nigeria is 41. Which of these is NOT an Island in Africa? Kwara state located? A. Fiji A. North east 15

Download More FREE Past Questions at www.prep.ng

B. North west B. sturdy C. North central C. Fearful D. South south D. strong E. East 57 Which of these United Nations Agencies is 50. 50% of a kilometre is ____. responsible for the promotion of culture? A. 500mm A. ILO B. 500m B. WHO C. 500cm C. UNESCO D. 500ft D. UNICEF E. 500yd 58.Feeble: 51. Ojo and Aina bought 25 oranges each and A. Strong gave Audu and Amina 6 and 8 oranges each B. weak from both. How many oranges will both of C. Tired them have left? D. Little A. 18 each B. 19 and 17 59. Permanent: C. 36 A. temporary D. 36 each B. old C. lasting Choose a word which has the same or D. complete nearly the same meaning as the first word. 60. Boss: A. Governor 52.Prohibit: B. Servant A. Advertise C. Mistress B. Allow D. master C. Forbid D. Destroy 61. Dear: A. precious 53. What type of vegetation are you likely to B. cheap find at the northernmost parts of Nigeria? C. nice A. Guinea savannah D. useless B. Sudan savanna C. Mangrove forest 62. Elegance: D. Rain forest A. grace E. Sahel savannah B. display C. safety Instructions: Choose a word which has the D. ugly same or nearly the same meaning as the first word. Use it to answer question 54-56 and 63. Thankfulness: 58-65 www.prep.ngA. greatness B. gratitude 54. Astonish: C. meanness A. Asunder D. goodness B. Attack C. Surprise 64. Study: D. Sensible A. Learn B. Enjoy 55. Miserable: C. Teach A. Mischief D. Remember B. Wretched C. Mistake 65. Excitement: D. Wicked A. Enthusiasm B. Zeal 56. Stubborn: C. Fuss A. obstinate D. Success 16

Download More FREE Past Questions at www.prep.ng

73. I do not like grapes or bananas but I have Complete the following simile: bought some fruits to eat. Do you think it is grapes, bananas, both or neither? 66.Her gown is as green as: A. Grapes A. grace B. Bananas B. grass C. Both C. flag D. Neither D. butter 74. There is a boat with a ladder attached to it 67. A grandfather is ones . and the ladder is eight metres tall if the water A father’s father only rises four metres how much of the ladder will B mother’s father be on top of the water? C both mother and fathers’ father A. 12 metres D. All fathers and mothers above 50years of B. 48 metres age. C. 8 metres D. 10 metres 68. Suleiman is taller than Ajayi while Usman is taller than Suleiman and Tope who is taller 75. I thought the time was ten minutes to two, than Suleiman. Who is the shortest? but I was looking at the clock in a mirror. What A. Usman time was it really? B. Suleiman A. ten minutes to two C. Ajayi B. ten minutes after two D. Tope C. ten to ten D. Ten to three 69. Some oranges were shared between two children in ratio three to five respectively. If ANALOGIES AND ASSOCIATIONS. Choose the first child’s share was 120, how many the correct word to complete the oranges were shared between them? analogies. Use it to answer questions 76-79. A. three hundred and twenty B. two hundred 76. Gloves are to hands as shoes are to ____. C. four hundred and twenty A. Boxers D. two hundred and twenty B. Hands C. feet 70. Increase six hundred by twenty five D. Legs percent A. one hundred and fifty 77. Up is to down as before is to: B. seven hundred and fifty A. later C. one thousand B. after D. six thousand C. now D. next 71. Bisi is older than Ore, Dele is older than Bisi. Who is the youngest of all? 78. Sew is to needle as dig is to: A. None www.prep.ngA. garden B. Bisi B. plant C. Ore C. poke D. Dele D. spade

72. Abayo, Tope, Femi and Seyi are friends. 79. School is to pupils, as army is to: Only Abayo likes going to the theatre. Only A. teachers Tope and Seyi like the cinema. Only Femi and B. chiefs Seyi like dancing. Which boy likes neither C. soldiers dancing nor cinema? D. barracks A. Tope B. Seyi Complete the following simile: C. Abayo D. Femi 80. Olu is as hungry as a: A. orphan B. wolf 17

Download More FREE Past Questions at www.prep.ng

C. termites D. Thug

ANSWERS TO 2008 POST-UTME QUESTIONS

1. C 2. D 3. B 4. B 5. D 6. C 7. D 8. B 9. D

10. B 11. C 12. D 13. B 14. C 15. C 16. D

17. B 18. D 19. C 20. D 21. B 22. C 23. B

24. C 25. A 26. A 27. C 28. D 29. B 30. A

31. B 32. B 33. C 34. B 35. C 36. A 37. B

38. D 39. C 40. B 41. A 42. B 43. D 44. B

45. D 46. B 47. B 48. C 49. C 50. B 51. A

52. C 53. E 54. C 55. B 56. A 57. C 58. B

59. C 60. D 61. A 62. A 63. B 64. A 65. A

66. B 67. C 68. C 69. A 70. B 71. C 72. C

73. D 74. C 75. B 76. C 77. B 78. D 79. D

80. B

www.prep.ng

18

Download More FREE Past Questions at www.prep.ng

UNILORIN POST UTME PAST QUESTIONS 2009/2010

1. Abuja officially replaced Lagos as capital of in the sentence. Use it to answer Nigeria in ______. question 9-13. A. 1992 B. 1996 9. Her y o u n g e r s is t e r likes conservative C. 1998 hair style but her cousin prefers ____ hairstyle D. 1991 A. Modest B. New 2. The highest court in Nigeria is the ______. C. Modern A. Federal High Court D. Magnanimous B. Supreme Court C. Federal Court 10. She adores eating junks but _____eating D. Sharia Court fruits. A. Loves 3. The ro tatio n of the e arth on it s a x i s B. Supports causes? C. Dislikes A. Autumn and Winter D. Disliked B. Wet and dry season C. Summer and water 11. The brexity of the first message contrasts D. Dawn and Twilight with the ______second message A. Shortness 4. The earth is the _____? B. Integrity A. Closest planet to the sun C. Verbosity B. 2nd closest to the sun D. Moral C. 3rd closest to the sun D. 4th closest to the sun 12. Humility is a virtue while------can be regarded as a vice 5. Naira is to Nigeria as ______is to India. A. Pride A. Yen B. Proud B. Francs C. Curiosity C. Rupee D. Position D. Dupee 13. Femi is very Presumptuous whereas his 6. Collection of different human organs friend is ______. working as a unit is referred to as ____. A. Kind A. Tissue B. Modest B. Cell C. Proud C. System D. Pompous D. Skeleton 14. An iron rod feels cooler in the hand than a 7. A body of shallow sea water or brackish wooden rod at the same temperature. The water separated from thewww.prep.ng sea by some form of reason is that iron------bamer is called _____. A. is a better conductor of heat than wood A. Nile’s Valley B. has a higher density than wood B. Lagoon C. has a higher expansivity than wood C. Greeks D. is heavier than wood of the same size D. Oasis 15. Which of the following is used for 8. HIV can be transmitted in which of these controlling the amount of light entering the ways except _____. eye? A. Homosexual intercourse A. Cornea B. Heterosexual intercourse B. Pupil C. Mosquitoes as a Vector C. Iris D. Needle pick injury with infected blood D. Ciliary muscles

Instruction: Choose the option that is 16. A labourer is paid N5,400:00k per month. 3 opposite in meaning to the underlined word What is his total salary in 1 /4years? 19

Download More FREE Past Questions at www.prep.ng

A. ₦113,40:00 A. Energy B. ₦64,800:00 B. Good sight C. ₦97,200:00 C. Good teeth D. ₦129,600:00 D. Strong bones

17. Which of the following will sink when 25. President Barrack Obama is the ____ placed on water? president of the United States of America and A. Ball first African- American president of U.S. B. Plastic Cup A.35th C. Balloon th D. Eraser B.44 C.40th 18. The following are domestic birds except D.39th A. Duck B. Pigeon 26. A large area of water surrounded by land is C. Turkey called______. D. Vulture A. A dam B. An Island 19. A boy buys a biscuit for N4 and sells it at a C. An Iceberg profit of 20%. Find the actual price and the D. A lake selling price? A. i =N1.00k, ii = N5.00k 27. A written piece of paper which indicates B. i = 60k, ii = N4.40 that goods bought and services rendered have C. i = 80k, ii = N4.80k been paid for is called_____. D. i = 50k, ii = N4.50k A. An Invoice B. A receipt 20. A man buys recharge card for ₦500. He C. A wage bill sells it for ₦430. What is the percentage loss? D. A trade document A. 7% B. 9% 28. The device for stepping up or down C. 14% electricity voltage on transmission line is D. 20% called______. A. Electricity meter 21. Under whose regime were Delta and B. Transmission station Adamawa States created? C. Transformer A. Gen Sani Abacha D. Step-down panel B. Gen Yakubu Gowon C. Gen Murtala Muhammed 29. The major raw material for pottery is D. Gen Ibrahim Babangida ______. A. Metal 22. The body charged with the responsibility of B. Ceramics regulating and protecting consumers against C. Cement illicit foods and drugs in Nwww.prep.ngigeria D. Clay A. Consumer protection council B. Standard Organization of Nigeria 30. If the total sales for a bag shop in a certain C. National Drug Law Enforcement agency year were ₦150, 000 What were sales in July, D. National Agency for Food, Drug if July sales were half the monthly average? Administration and Control A. ₦6,250 B. ₦8,650 23. The senate president during the second C. ₦12,500 republic was _____. D. ₦7,550 A. Dr. Joseph Wayas B. Senator Anyim Pius Anyim 31. If 15 cans of food are needed for seven C. Senator Iyorchia Ayu adults for two days, the number of cans D. Chief Adopous Wabara needed to feed four adults for seven days is? A. 25 24. We eat Carbohydrate food to give us B. 15 _____. 20

Download More FREE Past Questions at www.prep.ng

C. 20 A. 5 D. 30 B. 6 C. 4 32. Which one of the following key techniques D. 7 is necessary for all the others to be effective? A. Communication 41. The followings are true concerning a B. Teamwork trapezium except ____. C. Leadership A. It does not have any line of symmetry D. Technical know-how B. All the sides are of different lengths C. All the angles are of different sizes 33. ____ you come early to the new house, D. It has two angles equal but others different. clean up my flat. A. By chance 42. A man bought 100 eggs at N1500.00, 10 B. While eggs were damaged. He sold the rest at C. Should ₦20.00 each. Calculate the percentage profit D. Should in case made. A. 17 34. Those men duped Dupe and ______. B. 15 A. Myself C. 20 B. I D. 12 C. Me D. We 43. In a leap year a baker baked 500,000 loaves of bread, how many loaves did the 35. The physicians have more people in ___ baker bake in February, if his working days are waiting room than ____ have ever had. Mondays to Saturdays? A. His/he A. 1592 B. There/they B. 39800 C. Their/they C. 38208 D. Them/they D. 46168

36. It was Friday on Jan 1st, 2010. What was 44. In Nigeria democracy day is marked on the day of the week Jan 1st, 2011? ___. A. Sunday A. June 12 B. Wednesday B. May 29 C. Thursday C. June 29 D. Saturday D. May 27

37. Choose the odd one among the followings. 45. The ______originated the division of time A. Cougar as base of 60. B. Hyena A. Babylonians C. Cheetah B. Americans C. Leopard C. Romans www.prep.ngD. Arabs 38. Which among the following does not belong to the group, 46. In the computer browser address field, A. Brigadier what does the abbreviation www stands for B. Colonel _____. C. Navy Captain A. world wide web D. Group Captain B. word wide web C. web wide world 39. In which part of Africa will you find D. wide web world Tanzania A. West Africa 47. The earth is the _____ planet from the B. North Africa sun. C. Southern Africa A. 3rd D. East Africa th B. 4 40. A hexagonal kite has ___ sides. C. 2nd 21

Download More FREE Past Questions at www.prep.ng

th D. 5 B. C. Muritala Muhammed 48. The following deceases are transmitted by D. Ibrahim Babangida mosquitoes except _____. A. Yellow fever 56. Which is the odd one out among the B. Dengue fever followings? C. West Nile fever A. African National Congress D. Hepatitis fever B. Social Democratic Party C. National Republican Convention 49. In a coding system numbers 0 to 9 are D. All Peoples Party coded a to j respectively, what will be the code for the number 4238 57. The last military head of state in Nigeria A. ebdj was B. fdei A. Sanni Abacha C. ecdi B. Ibrahim Babangida D. dbch C. Abubakar Abdulsalam D. Muritala Muhammed 50. Obtain the missing number in the following sequence: 2, 3, 5, 8, __, 17, 23, 30 58. GSM telephony was introduced into Nigeria A. 13 communication system during the regime of B. 12 _____. C. 11 A. Ibrahim Babangida D. 14 B. Olusegun Obasanjo C. Umaru Yar’adua 51. A retailer sells at a discount of 15% of the D. Goodluck Jonathan marked price, if a customer pays ₦16,700.00 for an item, what is the marked price of the 59. The following Nigerian footballers have won item? the African Footballer Award except A. ₦19,650.00 A. Victor Ikpeba B. ₦13,750.00 B. Nwankwo Kanu C. ₦20,700.00 C. Rasheed Yekini D. ₦22,700.00 D. Austin Okocha

52. Given that G is directly proportional to the 60. Kofi Annan is well known for his role as square of H. If G is 5 when H is 3, Find H when _____. G is 100 A. President of Ghana A. 150 B. Secretary General of United Nations B. 125 C. Secretary General of the African Union C. 180 D. President of the United Nations D. 225 61. Complete the followings; Ooni and Osun, 53. Late Idi Amin Dada was from which of the Sultan and Sokoto, Obi and Anambra, El following countries ____.www.prep.ng Kanemi and ______. A. Kenya A. Kaduna B. Tanzania B. Borno C. Uganda C. Katsina D. Congo Brazzaville D. Yobe

54. River Nile has its source from ____. 62. In which part of Nigeria will you find relics A. Lake Chad of groundnut pyramids? B. Lake Victoria A. Kano C. Lake Tanganyika B. Jos D. Lake Albert C. Sokoto D. Maiduguri 55. Who was the head of state when Nigeria first won the African Cup of Nations. 63. Nnamdi Azikwe International Airport is A. Shehu Shagari found in ____. A. Enugu 22

Download More FREE Past Questions at www.prep.ng

B. Port Harcourt 72. An area of land almost surrounded by C. Abuja water is known as ______. D. Calabar A. Lake B. Lagoon 64. The time zone operative in Nigeria is ____. C. Island A. GMT D. Peninsula B. +1 GMT 1 20 C. – 1 GMT 73. Solve /4 of /7 3 D. +2 GMT A. /7 7 B. /5 5 65. The rotation of the earth on its axis is used C. /7 2 in the measurement of _____. D. /7 A. Time B. Weather 74. The cost of 2 biros and 3 pencils is ₦18, if C. Temperature a pencil cost ₦2.50k, how much is a biro? D. Pressure A. ₦4.25k B. ₦3.20k 66. The Nigerian currency in use before the C. ₦4.00 change to the Naira was the D. ₦5.25K A. Pound B. Dollar 75. Solve 21% of 70 C. Riyal A. 29.4 D. CFA B. 14.7 C. 13.9 67. An oncologist is a medical practitioner D. 7.4 specialized in the treatment of A. Tumours 76. Wale and Joke have ratio 6:4 in 80 units of B. Fibroids Transcorp’s shares. How many units of these C. Obesities shares belong to Wale? D. Ulcers A. 48 B. 40 68. Car is to road as train is to _____. C. 32 A. Wheels D. 24 B. Rails 2 1 C. Surface 77. The sum of and /3 and /3 is what? 1 D. Locomotive A. /3 2 B. /3 69. Which is the odd one out? C. 1 A. Hockey D. 3 B. Exercise C. Tennis 78. A trader had 19 lanterns in her store, sold D. Football 13 and bought 6 more. How many lanterns www.prep.ngdoes she left in her store? 70. A forest is to a tree as a tree is to a A. 6 ______. B. 9 A. Leaf C. 12 B. Orchard D. 4 C. Jungle D. Plant 79. Add all the even numbers from 15 to 20 A. 55 71. The non-resident head of a University is B. 34 ______. C. 54 A. Vice-chancellor D. 64 B. Chancellor C. Provost 80. Which of the following fractions is the D. Registrar highest? 3 A. /4 7 B. /9 23

Download More FREE Past Questions at www.prep.ng

2 C. /3 12 D. /15

ANSWERS TO 2009 UNILORIN POST UTME QUESTIONS

1. B 2. B 3. D 4. C 5. C 6. C 7. B 8. C 9. C

10. C 11. C 12. A 13. B 14. A 15. C 16. A

17.D 18. D 19. C 20. C 21. D 22. D 23. A

24. A 25. B 26. D 27. B 28. C 29. D 30. A

31. D 32. A 33. C 34. C 35. C 36. D 37. B

38. A 39. D 40. B 41. D 42. C 43. C 44. B

45. A 46. A 47. A 48. D 49. C 50. B 51. A

52. C 53. C 54. B 55. A 56. A 57. C 58. B

59. D 60. B 61. B 62. A 63. C 64. B 65. A

66. A 67. A 68. B 69. B 70. A 71. B 72. D

73. C 74. D 75. B 76. A 77. C 78. C 79. C

80. D

www.prep.ng

24

Download More FREE Past Questions at www.prep.ng

UNILORIN POST UTME PAST QUESTIONS 2010/2011

D. having sold Instruction: Complete the gap with the E. selling best option. 8. As ______of you as can come are welcome. 1. Peter’s ____ a cold upset our plan. A. each A. catches B. any B. will catch C. many C. to catch D. more D. to have caught E. most E. catching 9. He bade them ...... to his house. 2. Although he was invited ____ he was not A. welcomed welcome. B. welcome A. but C. to be welcome B. yet C. so D. to have been welcome D. since E. welcomes E. because 10. If ...... one day, we would talk about it 3. When John reported the incident, the for weeks. teacher remarked that he ...... a responsible A. a lion will be coming into this room B. a lion boy. was to come into this room A. was C. a lion came into this room B. is D. a lion is coming into this room C. will be E. a lion is going to come into this room D. should have been E. is going to be 11. Excuse me, do you mind _____ I smoke? A. if 4. They tried to cash in _____ the people's B. since ignorance. C. while A. under D. against B. against C. about 12. The house will look all the better ____ this D. with new coat of paint. E. on A. for B. under 5. A pioneer, he ____ the institution from his C. against own resources. D. upon A. found and funded E. over B. founded and funded C. was finding and funding 13. Without _____ words he accused him D. had fund and funded www.prep.ngdirectly of treachery. E. was founding and funding A. amending B. modifying 6. He passed so well ____ he was awarded a C. mixing scholarship. D. mincing A. since E. meaning B. because C. that 14. He is easily the ____ of the lot. D. while A. brightest E. thus B. brighter C. more bright 7. Government should prevent smugglers D. most brightest _____ their contraband. E. most bright A. to sell B. to be selling 15. I am not attending _____ is my wife C. from selling A. whereas 25

Download More FREE Past Questions at www.prep.ng

B. neither D. I wanted her to buy anything C. either E. she wanted her to buy anything D. but E. since 23. He devoted himself _____ homeless children. 16. Some people think that ...... are A. to helping potentially a nuisance. B. to help A. Parent-in-law C. by helping B. Parents-in-law D. with helping C. Parents-in-laws E. helping D. Parents-ins-laws E. Parent-in-laws 24. Although the problem was simple ______students were able to solve it. 17. Shakespearean _____ are generally the A. little victims of circumstances. B. Small A. heros C. few B. heres D. a few C. heroses E. a lot of D. hero E. heroes 25. We walked quietly _____ wake the baby. A. so that not 18. _____ a good film, wasn’t it? B. in order to A. It is C. so as to B. Was it D. so as not to C. There is E. so not to D. It was E. There was 26. These six employees _____absent from work since New Year's Day 19. We left _____USA and crossed _____ A. are Atlantic Ocean to_____ Europe. B. have been A. the/the/...... C. were B...... /..../.... D. had been C...... /..../the E. are being D. the/...../a E...... /an/...... 27. I wish I ...... to swim when I was younger. A. had learnt 20. There was ____ much noise at night that B. learn we couldn’t sleep C. learnt A. such a D. was learning B. such E. have learnt C. that D. very 28. By the end of June, I ...... six E. so www.prep.ngexaminations this year. A. shall take 21. Have you paid your taxes up to date? The B. have taken tax collector inquired if _____. C. shall have taken A. You have paid your taxes up to date D. am taking B. I had paid my taxes up to date E. took C. I paid my taxes up to date D. have I paid my taxes up to date 29. We used to go to that club on Saturday E. I have paid my taxes up to date night, ____? A. Isn’t it 22. Do you want me to buy her anything? She B. didn’t we asked Asman. She asked Asman If C. don’t we _____. D. aren't we A. she wants her to buy her anything E. used we not B. you want to buy anything C. she wanted to buy anything 26

Download More FREE Past Questions at www.prep.ng

Choose the antonyms nearest to the D. funny underlined word. Use it to answer question 30-39. 38. Chibuzor gave a curt nod and walked away A. rude 30. Oche entered the principal's office in a B. polite rather abrasive manner C. shocking A. gentle D. gentle B. rude C. lackadaisical 39. The girl took a cursory glance at the letter D. Indifferent and hid it A. concise 31. Otokpa is a member of the ad-hoc B. brief committee on stock acquisition C. lasting A. improvised D. sententious B. formal C. temporary Choose the antonyms nearest to the D. fact-finding underlined word. Use it to answer question 40- 49. 32. His gift to the poor was always infinitesimal A. large 40. The cynics feared that the nation's nascent B. small democracy would fail. C. supportive A. Pessimists D. shameful B. delinquents C. critics 33. The economist concluded that several D. illusionists factors have been adduced to explain the fall in the birth rate 41. The essence of governance is to seek the A. affirmed good and well-being of the majority of the B. diffused people C. mentioned A. importance D. refused B. goal C. characteristics 34. The presidential system is to an antidote D. secret some political ailments A. an answer 42. From what she said, one may infer that he B. a reply does not like the course. C. an inquiring A. suppose D. an obstacle B. realize C. deduce 35. Ola thought that her father was very D. agree callous A. parlous 43. He shared his room with a person whose B. compassionate www.prep.ngbehaviour was quite nauseating C. wicked A. disrespectful D. cheerful B. disgraceful C. discouraging 36. He was very much respected, though he D. disgusting had no temporal power. A. spiritual 44. The carpenter built a commodious B. Mundane wardrobe. C. permanent A. gigantic D. ephemeral B. small C. spacious 37. The way the workshop was organized was D. wide rather hit-and-miss. A. systematic 45. Publishing as a business venture has B. hasty become a hot potato in Nigeria C. slow A. unpleasant 27

Download More FREE Past Questions at www.prep.ng

B. profitable 54. Avian Flu is a disease in which of the C. unacceptable following? D. expensive A. Mammals A. Birds 46. The man's story sounded plausible to his C. Pisces audience. D. Amphibians A. fantastic B. credulous 55. Which of the Following is not an excretory C. credible product? D. entertaining A. Sweat B. Urea 47. The presence of the captain makes the C. Urine sailors ill at ease D. Dust A. impatient B. easily ill 56. What is the Angle in degrees on a straight C. uncomfortable Line? D. sickly A. 90˚ B. 60˚ 48. Ibro shows enough liberality with his C. 180˚ meagre income D. 30˚ A. insensitivity B. prodigality 57. Convert 0.005 kilograms To Grams. C. frugality A. 5000 D. generosity B. 500 C. 50 49. It is a misnomer to call three thousand D. 5 naira a living wage A. a wrong description 58. What Is the value of DCXL. B. an incontrovertible assertion A. 640 C. an appropriate term B. 504 D. a mishmash C. 540 D. 600 50. A Place Where Two Rivers Meet is Called A. Confluence 59. A number of people listening to a B. Junction concert. C. Joint A. congregation D. Delta B. spectators C. audience 51. The Primary Source of Energy is called D. crowd _____. A. Sun 60. A number of singers in a church. A. Water A. troupe C. Food www.prep.ngB. choir D. Heat C. entertainers D. artists 52. The first colour Of the Rainbow is called____. 61. A number of sailors manning a ship. A. Green A. crew B. Violet B. gang C. Indigo C. staff D. Red D. host

53. Which of the Following Is Not A Mammal? 62. A number of directors of a company. A. Whale A. panel B. Dog B. jury C. Cat C. bench D. Shark D. board

28

Download More FREE Past Questions at www.prep.ng

1 3 5 1 63. A number of persons of the same race C. /8, /7, /9 and /34 1 1 3 5 and character is ____. D. /8, /3, /7 and /9 A. gang B. tribe 72. Arrange the following in descending order 2 4 5 1 C. associates /3, /7, /9 and /4 2 4 5 1 D. fellow A. /3, /7, /9 and /4 1 5 4 2 B. /4, /9, /7 and /3 1 4 5 2 64. A place where milk is converted into C. /4, /7, /9 and /3 1 2 4 5 butter and cheese is ___. D. /4, /3, /7 and /9 A. dairy B. bakery 73. What is Roman numeral of 50? C. distillery A. L D. refinery B. X C. C 65. A place for housing aero planes is called D. D ____. A. tarmac 74. What is Roman numeral of 19? B. garage A. XX C. hangar B. IX D. airport C. XIX D. D 66. A place where government records are kept _____. 75.What is Roman numeral of 60? A. library A. XL B. archive B. LX C. government house C. CX D. museum D. XD

67. A place for storing grain is called ____. 76. What is Roman numeral of 1000? A. depot A. D B. store B. L C. bans C. C D. granary D. M

68. The 2010 world cup was hosted by _____. 77. What is Arabic figure of Roman figure IC? A. Brazil A. 49 B. South Africa B. 99 C. Germany C. 101 D. Argentina D. 89

69. The 2008 Olympic was held in which of 78. What is Arabic figure of Roman figure ID? these cities? A. 499 A. Berlin www.prep.ngB. 299 B. Greece C. 401 C. Beijing D. 489 D. London 79. What is Arabic figure of Roman figure VC? 70. The 1998 world cup was hosted and A. 99 won by _____. B. 95 A. Brazil C. 105 B. France d. 500 C. England D. USA 80. Subtract 6289 from 4389 A. 1900 71. Arrange the following in ascending order B. 1800 1 3 5 1 /8, /7, /9 and /3 C. -1900 1 3 5 1 A. /8, /7, /9 and /3 D. -1800 3 1 5 1 B. /7, /8, /9 and /3 29

Download More FREE Past Questions at www.prep.ng

ANSWERS TO 2010 POST UTME QUESTIONS

1. E 2. B 3. A 4. E 5. B 6. C 7. C 8. C

9. B 10. C 11. A 12. D 13. D 14. A 15. B

16. B 17. E 18. D 19. A 20. E 21. B 22. E

23. A 24. C 25. D 26. B 27. A 28. C 29. B

30. A 31. B 32. A 33. D 34. D 35. B 36. A

37. A 38. B 39. A 40. A 41. A 42. C 43. D

44. C 45. A 46. C 47. C 48. D 49. A 50. A

51. A 52. D 53. D 54. B 55. D 56. C 57. D

58. A 59. C 60. B 61. A 62. D 63. B 64. A

65. C 66. B 67. D 68. B 69. C 70. B 71. D

72. A 73. A 74. C 75. B 76. D 77. B 78. A

79. B 80. C

www.prep.ng

30

Download More FREE Past Questions at www.prep.ng UNILORIN POST UTME PAST QUESTIONS 2011/2012

1. The judge restrained the landlord D. adoption from_____ the tenant. E. kidnap A. sacking B. ejecting 8. There was a ten-minute ...... during the C. dismissing stage performance of the play D. disqualifying A. interim B. interview 2. The university awarded _____ doctoral C. interlude degrees to four distinguished members of the D. interplay community. E. intercession A. honourable B. expensive 9. The judge warned that he would not tolerate C. ceremonious any interruption of the ...... of the court. D. ceremonial A. proceeds E. honorary B. deliberations C. debates 3. The students threatened to embark on D. proceedings a______of lectures if their demands were not E. processions met. A. condonation 10. The satellite maintains a constant ...... B. suspension round the earth. C. rejection A. path D. strike B. orbit E. boycott C. circle D. route 4. The kidnappers demanded a ____ of E. chart N50,0000 before they would release the rich businessman. 11. I want to know your religious _____. A. levy A. believe B. ransom B. believing C. penalty C. beliefs D. freedom D. believes E. condition 12. He runs _____. 5. The government scolded the contractors for A. rather much fast the poor _____ of the project. B. quite rather fast A. prosecution C. rather too fast B. investigation D. too rather fast C. execution D. installation 13. The previous assignment was _____for E. establishment www.prep.ngme. A. much too difficult 6. The new car is strikingly attractive but I B. too much difficult very much doubt its ____ C. difficult too much A. durability D. much difficult too B. style C. harmony 14. The TV set, ...... was damaged last week, D. longevity had been taking to the electronics engineer E. complexity A. which B. who 7. Since there were no further comments the C. of whom chairman called for the ...... of the minutes of D. of which the last meeting A. objection 15. My _____house is by the side of the road. B. resolution C. admission A. father-in-law's 31

Download More FREE Past Questions at www.prep.ng

B. father's-in-law C. with C. fathers'-in-law D. for D. father-in-law 24. Stella wanted to show _____ with her 16. The students were advised to look ____ necklace. difficult words in the dictionary A. off A. in B. on B. on C. over C. out D. back D. up 25. The woman _____ daughter he married, is 17. He was charged with complicity _____the his mother-in-law. abortive coup. A. whose A. in B. which B. for C. her C. about D. whom D. on 26. He acts as if he ____ a general manager. 18. He was convicted ____ stealing some bags A. is been of cocoa. B. were A. with C. has been B. in D. is being C. of D. for 27. The national essay competition came

_____ on the 23rd of July 1986 19. Mary said she was acting ___ the A. out instructions of the principal. A. by B. in C. by B. on D. up C. for

D. through 28. Mrs Nwokolo had ...... her breakfast before

20...... normal circumstances, it is rare to her friend came in A. has get all registered voters to vote. B. have

C. had A. on D. been had B. by C. in 29. If the two boys ___ been bitten by the D. under snake, they should be taken to the hospital.

A. has 21. Salary cuts could be the ____ of the B. had worker's protest. A. course C. have www.prep.ngD. is B. curse

C. cause 30. The man _____ missed death during the D. coarse collision.

22. Neither John nor Mary felt happy ____the A. extensively B. always incident. C. narrowly A. for D. amply B. in

C. about D. by 31. I started writing at 9am. It is now 10. By 11am, I ___ writing for 2 hours.

A. will be 23. Any parent would be pleased _____ such B. am impressive performance. C. will have been A. at D. have been B. from 32

Download More FREE Past Questions at www.prep.ng

B. too old 32. My ...... to you, students, is to be good C. older enough A. advice D. old enough B. advise C. adverse 41. We won’t leave until it ____ raining. D. advances A. will stop. B. stopped 33. The car couldn’t keep pace ____ the motor C. stops cycle especially in congested areas. D. has stopped A. along B. with 42. The ...... event marked the beginning of a C. down new life for the entire cabinet. D. on A. historical B. historicity of the 34. I want everybody to learn the last ten lines C. historic of the poem ___ heart. D. history of the A. with B. in 43. The train ____ before I arrived. C. for A. was leaving D. by B. has left C. had left 35. They went ___ talking after midnight D. would leave despite the fact that they all had to get up early. 44. You can travel on your own, _____? A. by A. Isn’t it B. for B. can’t you C. on C. won’t you D. with D. shan’t you

36. They are always scared to visit Mr. Ade 45. The man is refurbishing the flat with a view because he is ____. to ____ it. A. an army A. sell B. a soldier B. selling C. an army man C. have sold D. a military D. be selling

37. I want that spoon washed, I can still see 46. The Secretary General was shocked at the ..... on it. number of ____. A. oilness A. child's soldiers B. grease B. childish soldier C. some oil C. children soldiers D. greasiness D. child soldiers 38. Everybody has to do www.prep.ng...... homework. A. his 47. In spite of Shade's good looms, her hair is B. them always ____. C. theirs A. unkept D. there B. unkempt C. unwashed 39. Having worked all day, it is about time we D. uncared for ____ back home. A. go 48. By the end of the football match today, we B. goes _____the best player. C. went A. could have known D. gone B. might have C. will have known 40. At eighteen, Bose is too _____ to take D. would have known good care of her sisters. A. enough 33

Download More FREE Past Questions at www.prep.ng

49. It is African for a younger person to show 57.A trained person that specializes in the _____ to elders. treatment of diseases and disorders of the A. understanding teeth is called _____. B. indifference A. Cardiologist C. deference B. Dermatologist D. satisfaction C. Physician D. Dentist 50. A nursery rhyme is used to teach pupils how to spell the word. 58.A bricklayer is Paid ₦1000:00k for 5days A. hipoppotamus work. What is his pay for 22 days work? B. hippopotamus A. ₦22,000:00k C. hipopotamus B. ₦50,000:00k D. hippoppotamus C. ₦4,400:00k D. ₦5,000:00k 51. The title of the traditional ruler of Benin is called ____. 59. In parliamentary system of government, A. Obi members of the parliament are required to B. Obong report the proceedings of the house to their C. Oba _____. D. Igwe A. Local government chairman and governors B. Constituencies 52. Which of these is not a method of C. Political parties and party stalwarts preventing iron from rusting? D. Traditional rulers A. Lubrication B. Galvanization 60. One essential duty of a citizen to his/her C. Melting state is to ____. D. Painting A. Vote and support the government in power B. Recite the national anthem and the pledge 53.Which of the following is not a rainbow C. Pay his tax colour? D. Encourage other citizens to be loyal A. Indigo B. Purple 61. Today is Monday. After 63 days, it will be: C. Orange A. Wednesday D. Green B. Monday C. Tuesday 54. A person who flies an aircraft is called D. Saturday _____. A. A captain 62. At 10% discount, an item valued at B. A craftologist ₦45000 now cost ___. C. A pilot A. ₦4050 D. An aeronautics engineer B. ₦40500 C. ₦40505 55.Kwame Nkrumah Muswww.prep.ngeum is located in D. ₦40550 ____. A. Tanzania 63. In which country is Mississippi river B. Kenya A. USA C. Mauritania B. Argentina D. Ghana C. Brazil D. Canada 56.An example of confluence town in Nigeria is _____. 64. On a road map, a line segment of 4 inches A. Onitsha long represents a distance of 12 miles. How B. Obudu many miles long is 4.5 inches. C. Lokoja D. Port-Harcout A. 12.5 B. 13.5 C. 14.5 D. 15.5 34

Download More FREE Past Questions at www.prep.ng

73. How many hours and minutes are there 65. The breadth and length of a rectangle are L from 7.45am to 3.30p.m.? and 2L respectively. What is the perimeter of A. 6hrs. 45mins. the rectangle? B. 8hrs 30mins A. L C. 7hrs 45mins B. 3L D. 7hrs 30mins C. 5L D. 6L Complete the following simile:

66. The place where water is found in the 74. As merciless as a: desert is called ____. A. grave A. lake B. grasshopper C. Catastrophe D. Catapult B. dam C. diff 75. Complete the following series: 1, 3, 5 ,7 , D. oasis 9, ___ ? A. 8 67. Which of the following has the highest B. 11 population in the world? C. 12 A. India D. 13 B. Russia C. China 76. Complete the following series: .. 2, 2, 4, 6, D. USA 10 ,? A. 14 68. The scientific way of referring to the B. 10 activity of the ear is ____. C. 20 A. audio visual D. 16 B. audio C. visual 77. Complete the following series: 0.1, 0.3, D. chemoreceptor 0.5, 0.7, 0.9,___? A. 1.1 69. The upper legislative arm of government B. 0.11 in Nigeria is headed by ____. C. 11 A. speaker D. 0.011 B. president C. leader 78. Complete the following series: 0, 3, 8, 15, D. secretary 24? A. 36 70. The degree of hotness and coldness of a B. 30 place is called ___. C. 35 A. rainfall D. 31 B. humidity C. temperature 79. Complete the following series: 4, 5, 7, 11, D. weather www.prep.ng19? A. 33 71. How many seconds do have in a day? B. 27 A. 3600sec C. 37 B. 86.4x 102 sec D. 35 3 C. 86.4x 10 sec 80. Complete the following series: 2, 5, 11, D. 864sec 23, 47? A. 80 72. How many seconds make one week? B. 95 A. 86.4x 104secs C. 92 B. 604.8x 103secs D. 101 3 C. 60.4810 secs D. 6864 secs

35

Download More FREE Past Questions at www.prep.ng

ANSWERS TO UNILORIN 2011 POST-UTME

1. B 2. E 3. E 4. B 5. C 6. A 7. D 8. C

9. D 10. B 11. C 12. C 13. A 14. A 15. A

16. D 17. A 18. D 19. B 20. D 21. C 22. C

23. C 24. A 25. A 26. B 27. D 28. C 29. C

30. C 31. C 32. A 33. B 34. D 35. C 36. B

37. C 38. A 39. C 40. D 41. C 42. C 43. C

44. B 45. B 46. D 47. B 48. D 49. C 50. B

51. C 52. C 53. B 54. C 55. D 56. C 57. D

58. C 59. B 60. C 61. B 62. B 63. A 64. B

65. D 66. D 67. C 68. B 69. B 70. C 71. C

72. B 73. C 74. A 75. B 76. D 77. A 78. C

79. D 80. B

www.prep.ng

36

Download More FREE Past Questions at www.prep.ng

UNILORIN POST UTME PAST QUESTIONS 2012/2013

Complete each of the following sentences D. Kalu Uche by choosing the option that most suitably E. Ikechuckwu Uche fills the space. Use it to answer question 1-2. Instruction: Complete each of the following sentences by choosing the 1. Awolowo is dead, ____? option that most suitably fills the space A. Is he Use it to answer question 8-9. B. was he C. Isn’t he 8. When the beggar was tired, he ____ down D. wasn’t he by the road side. A. Lied 2. Neither Agbo nor his parents ____ the B. Laid meetings now. C. Layed A. attended D. Lay B. attends E. Lain C. has attended D. attend 9. He did not like _____ leaving the class early. Find the nearest in meaning to the underlined A. We word(s). B. Us C. Our 3. The result of his experiment represents a D. Ourselves breakthrough in medical science E. Our selves A. An outstanding success B. catastrophe Instruction: Find the nearest meaning to the C. an end to such experiments underlined words. Use it to answer question D. a breaking point 10-13. E. a colossal failure 10. His jail terms were to run concurrently. 4. Which of these is correctly spelt? A. Simultaneously A. lenght B. uniformly B. lentgh C. Laboriously C. length D. Consecutively D. lenghtt E. Judiciously E. lengtt 11. Only the small-fry get punished for such 5. Must you say "good morning" to everyone in social misdemeanours. the street? No _____. A. Small boys A. I needn't B. Unimportant people B. I mustn't C. Frightened people C. I can't www.prep.ngD. Frivolous people D. I do not need E. Inexperienced people E. I will have to 12. The traffic situation in Lagos can lead to 6. Kumasi is in which of these countries disastrous consequences; a man lost a very A. Morocco lucrative job because he was held up by it. B. Senegal A. Confused C. Rwanda B. Delayed D. Niger C. Annoyed E. Ghana D. Intrigued E. Obstructed 7. Who scored Nigeria's goals in 2010 world cup in South Africa? 13. The state government appointed a A. Obafemi Martins commission of inquiring to go into the B. Nwankwo Kanu community's complaints. C. John Obi Mikel A. Investigate 37

Download More FREE Past Questions at www.prep.ng

B. Search A. Used C. Look for B. Are used D. Account for C. Were used E. Ascertain D. Use

14. Granted you are my senior by one year, 21. I have already _____the picture on the you cannot ...... me around as if I were your sitting room wall. servant A. hanged A. Control B. hunged B. Dribble C. hang C. Order D. hung D. Ask E. Compel 22. Adika ____ a message from the club regularly. 15. Opposite in Meaning A. Receives B. Had received The government is making determined C. Receive efforts to eradicate illiteracy D. Has received A. Compulsory B. Unreliable 23. The board has been having ...... series of C. Innocent meetings lately. D. Ineffective A. Some B. a 16. Opposite in meaning C. a lot of Sola's car is badly damaged; he has to look for D. Many an expert mechanic to fix it. A. Uneducated 24. _____ Lawyers enrolled at the Nigerian B. Amateurish Bar last year ____ this year. C. Awkward A. As much/As D. Scientific B. Very many/than C. More/ than 17. Opposite in meaning D. As many/ than The judge blamed the plaintiff for misleading the court. 25. I'm sorry I can't give you any of the A. Complainant oranges. I have ...... left B. Accused A. Few C. Defendant B. Little D. Prosecution C. only a little D. A few Complete each of the following sentences by choosing the option that most suitably fills 26. I can't offer you another loan, I have the space. Use it to answer question 18-37. ____money left in my account. www.prep.ngA. A few 18. I look forward to ...... you next week B. a little A. seen C. Little B. seeing D. few C. be seeing D. see 27. Do you think all _____ not enough to help with the investigations? 19. _____ told of his impending arrival l A. These informations are worked hard to make his short stay very B. This informations is comfortable. C. This information is A. Have being D. These informations are B. Having been C. Have been 28. The police came early enough to____ D. Having being bomb planted by the rioters. A. Diffuse 20. The villagers _____ to grow rice. B. Insulate 38

Download More FREE Past Questions at www.prep.ng

C. Detonate 37. One of the girls who ____ Impregnated D. Defuse _____ died. A. Was / have 29. The visitor was very uncomfortable B. Were / have because of his ____ nose. C. Was / has A. Runny D. Were / has B. Running C. Watery 38. My MOTHER served rice and fresh fish D. Flowing stew for dinner. A. What kind of meal did your mother serve for 30. I have the ____ of meeting him. dinner? A. Privilege B. Did your mother serve rice and fresh B. Privilege stew for lunch? C. Priviledge c. What kind of stew did your mother serve for D. Previledge dinner? d. Who serve a rice and fish stew for dinner? 31. All God's prophets were given the great ______. 39. Although he is in all respects a poor A. Commision student, he has managed to buy a tape B. Commition recorder to improve his ____ of French. C. Comission a. mastering D. Commission B. Ideas C. speaking 32. The attitude of my students to _____ D. Thinking baffles me. E. mastery A. Pronunciation B. Pronounciation 40. Invariably he ends all his letters "...... C. Pronouceation amicably” D. pronounceation A. You B. Your's 33. Each of the houses _____ a new look C. Yours' A. Have got D. Yours B. Have E. Your own C. Has D. Were given 41. We should take care _____ the robbers come back. 34. Four weeks_____ enough for the police to A. May be conclude their investigation. B. Should in case A. Were C. Perhaps B. Is D. Probably C. Are E. In case D. Has been www.prep.ng42. The man no longer feared his opponent 35. The quality of your questions which...... after their spokesman had assured him that always attracted attention ...... never been they were well disposed _____ him. in doubt A. To A. Have / has B. Around B. Have / have C. About C. Has / has D. Towards D. Has / have E. For

36. The police claim that a number of stolen 43. Our plane was scheduled to _____ at cars ______recovered. 10:00 hrs but it was delayed because of bad A. Have been weather. B. Has been A. Fly out C. Has being B. Take off D. Have being C. Start out D. Shoot off 39

Download More FREE Past Questions at www.prep.ng

E. Lift off B. Chemical C. Chairman 44. The frightening explosion in the D. Champagne factory_____ whole wing. A. Washed out 52. Three quarters of the people in the B. rooted out village ____ killed, but only half of their huts C. Wiped out ____affected. D. Flushed out A. were/was E. Cleared out B. Were/were C. Was/were 45. 2008 Olympic games was held in _____. D. Was/was A. Ghana B. Greece 53. The sergeant spoke to me in a _____ C. Nigeria manner. D. France A. Coerce E. China B. Coarse C. Course 46. A man buys recharge card for ₦500. He D. Casual sells it for ₦430. What is the percentage loss? A.7% 54. I shall find time for my ...... when I get B.9% ...... with this difficult assignment. C.14% A. Past-time/over D.20% B. Pass-time/over C. Pastime/through 47. President Barrack Obama is the president D. Passtime/ through of the United States of America and first African-American president of U.S. 55. Cairo is to Egypt as _____ is to Senegal. A.35th A. Bangul B.44th B. Monrovia th C. Dakar C.40 D. Yaoundé D.39th 56. Lusaka is a capital city of _____. 48. A place where two rivers meet is called a A. Tunisia _____. B. Uganda A. Joint C. Zambia B. Junction D. Zimbabwe C. Confluence D. Delta 57. Which mountain is the tallest in the world. A. Everest 49. The Managing Director did not pay his B. Kilimanjaro staff last month ____? C. Elbrus a. Didn't he www.prep.ngD. Blanc B. Had he not C. Is not it 58. What is the full meaning of ATM? D. Did he A. Automatic teller machine E. Has he B. Authentic teller machine C. Argument teller machine 50. If you would do me this favour, I _____be D. Automated teller machine very grateful. A. Will 59. The young of a horse is called ____. B. Shall A. foal C. Would B. ewe D. Should C. herd D. Stallion 51. Rhymes E. donkey Chalet A. College 40

Download More FREE Past Questions at www.prep.ng

60. _____ is used for the measurement of 68. Divide 22 by half and deduct 4. What do atmospheric pressure. you have? A. Thermometer A. 20 B. Barometer B. 30 C. Hydrometer C. 40 D. Sphygmomanometer D. 7

61. One of the following is used in measuring 69. Five years ago, Ade was 8 years old. How wind speed. old will Ade be in Ten years’ time? A. 18 A. Wind speed B. 13 B. Barometer C. 23 C. Thermometer D. 28 D. Anemometer 70. Which of the following fraction is the 62. Patience, Abdul Rahman and Tola received smallest? 1 three hundred and ninety-six naira to share in A. /2 1 the ratio of 5:4:3 respectively, how much did B. /3 1 Patience receive? C. /6 1 A. ₦132 D. /5 B. ₦198 C. ₦99 71. If you don’t want to ...... your car to D. ₦165 robbers, then don’t travel in the night. A. Loose 63. If 125 men can do a piece of work in 120 B. Lose days, how many men would be sufficient to do C. Lose the same work in 109 days D. Lost A. 115 men B. 96 men 72. When the driver lost control of his vehicle, C. 180 men the pedestrians began to run for ____. D. 150 men A. their dear lives B. their dear life 64. Shade went out with some money. She C. dear lives 5 spent /10 of it and had ₦2.40 remaining. How D. dear life much did she have before going out? A. ₦4.80 73. My uncle is one of the ...... of the B. ₦2.60 society. C. ₦6.00 A. Elitists D. ₦15.60 B. Elites C. Elite 65. Increase ₦3,400 by 13%? D. Elitist A. ₦442 B. ₦3,842 74. Atuk without giving any hint as to what he C. ₦3,413 www.prep.ngintended to do, went and ...... himself. D. ₦3,952 A. Hung B. Hang 66. I have 25 Oranges. I gave 10 to Ade and 5 C. Hanged to Bola. What fraction do I have remaining? D. Hunged A. 3/5 B. 2/5 75. Although the manager is busy right now, C. 4/5 he will be with you ...... D. 1/5 A. Immediately B. Soon 67. How many times is 0.04 contained in 44? C. Without delay A. 1.76 D. Right away B. 110 C. 1100 76. She usually works hard but ...... she could D. 11 be lazy A. At times 41

Download More FREE Past Questions at www.prep.ng

B. Atimes C. At times D. At time

77. I would like to ...... my father to Kaduna. A. Accompany B. Follow C. Escort D. Join

78. The man declared his intention from the _____. A. Onset B. Inset C. Outset D. Offset

79. Audi fought with his _____ brother. A. Senior B. Older C. Elder D. Bigger

80. The loquacious young lad reeled off all the names of the players _____. A. Off head B. At hand C. At heart D. Off hand

ANSWERS TO UNILORIN 2012

POST-UTME

1. C 2. D 3. A 4. C 5. A 6. E 7. D 8. D 9. C

10. A 11. B 12. B 13. A 14. C 15. D 16. B

17. C 18. B 19. B 20. A 21. D 22. A 23. B

24. C 25. A 26. C 27. C 28. D 29. A 30. B

31. D 32. A 33. C 34. B 35. A 36. A 37. D www.prep.ng

38. D 39. E 40. D 41. E 42. D 43. B 44. -

45. E 46. C 47. B 48. C 49. D 50. D 51. D

52. B 53. D 54. C 55. _ 56_ 57. A 58. D

59. + 60. B 61. D 62. D 63. 64. A 65. B

66. B 67. C 68. C 69. C 70. C 71. B 72. D

73. C 74. C 75. B 76. A 77. A 78. C 79. C

80. D

42

Download More FREE Past Questions at www.prep.ng UNILORIN POST UTME PAST QUESTIONS 2013/2014

D. Eagle 1. I cannot ____ any noise _____. A. Hare/here 10. The Apex Bank in Nigeria is ____. B. Ear/Hair A. Zenith Bank C. Hear/here B. First Bank D. Hear/hear C. Intercontinental Bank D. Central Bank 2. What does one good turn deserve? A. Other 11. The highest rank of an officer in the B. Another Nigerian Navy is ____. C. Each other A. Commodore D. One another B. Commander C. General 3. What will a rolling stone never gather? D. Admiral A. Rose B. Mass 12. AU stands for _____. C. Moss A. American Union D. Boss B. Africa United C. African Union 4. What is the mother of invention? D. African Unionism A. Difficulty B. Necessary 13. Which of these enzymes is responsible for C. Necessity the digestion of fat? D. Necessitate A. Lipase B. Pepsin 5. What should you make while the sun shine? C. Amylase A. Gay D. Cellulase B. Say E. Xylanase C. Day D. Hay 14. The marked price of a book is ₦180.00. E. Ray What will be the cash price if a discount of 6% is allowed? 6. What should you do before you leap? A. ₦210.80 A. Listen B. ₦30.60 B. Hear C. ₦169.20 C. Think D. ₦190.80 D. Look 216 15. Reduce /324 to its lowest terms. 1 Instruction: Complete the following idioms in A. /9 2 question 7-9. B. /3 3 C. /4 4 7. It takes a _____ catchwww.prep.ng a thief. D. /5 A. Gun B. Police 16. A ship sails 400km in 18hours. How many C. Soldier days will it take to cover 1600km at the same D. Thief rate? A. 3 days 8. What should you let sleeping dogs do? B. 4 days A. Lay C. 6 days B. Sleep D. 8 days C. Lie E. slide 17. By selling an article for ₦47.00, a man lost 6%. How much did he pay for it? 9. Who catches the worm? A. ₦25 A. Early bird B. ₦5.00 B. Late Bird C. ₦50.00 C. Early rise D. ₦53.00 43

Download More FREE Past Questions at www.prep.ng

C. 164 18. Shade went out with some money. She D. 200 5 spent /10 of it and had ₦2.40 remaining. How much did she have before going out? 26. The cost of electricity is 15K per unit. How A. ₦4.80 many units did a man consume if he paid B. ₦2.60 ₦42.55K C. ₦6.00 A. 283.67 D. ₦15.60 B. 2.837 C. 482.67 19. Add together the nine smallest numbers D. 48.27 from 1 to 19 A. 30 27. Whose picture is shown on 100 naira note? B. 55 A. Tafawa Balewa C. 42 B. Obafemi Awolowo D. 45 C. Nnamdi Azikiwe D. Herbert Macaulay 20. How many seedlings must be planted to obtain 60 trees if only 20% of the seedlings 28. The first woman to drive a car in Nigeria is survive? _____. A. 150 A. Mrs. Dora Akunyili B. 125 B. Mrs Funmilayo Ransome Kuti C. 300 C. Mrs. Elam Adebayo D. 250 D. Mrs. Juliana Coker

21. If your father has two wives, the daughter 29. The Nigeria female football Team is called of the other wife is your _____. A. Sister A. Flying eagles B. Half sister B. Super falcons C. Cousin C. Golden eagles D. Step sister D. Oladimeji queen

22. Dog, cat, goat and ______are examples of 30. Whose picture is shown on the five naira mammals. note? A. Snake A. Sir Tafawa Balewa B. Lizard B. Chief Anthony Enahoro C. chicken C. Sir Arthur Richards D. Whale D. Sir Ahmadu Bello

23. Amaka scored 60% in a test that had 50 31. The male part of a flower is called ____. questions. How many questions did she get A. pistil correct? B. corolla A. 25 C. stigma B. 30 www.prep.ngD. stamen C. 35 D. 40 32. The _____ is referred to as the pumping station of an animal. 24. In what ratio will 56 oranges be shared A. brain between Femi and Dayo such that Femi gets B. lung 16 and Dayo gets 40? C. heart A. 1:2 D. kidney B. 1:5 C. 2:5 33. The bone of the brain is covered by the D. 3:5 _____. A. patella 25. A farmer has 120 birds. If he has increase B. skull of 20% how many birds have he? C. tibia A. 124 D. scapular B. 144 44

Download More FREE Past Questions at www.prep.ng

34. Digestion of food starts from the ______. 42. I can say that our governor is angling for a A. mouth second from his party. B. anus A. The governor is rejecting a second term C. stomach B. The governor is indirectly asking for a D. gullet second term C. The governor is being asked to run a second 35. The missionary that stopped the killing of term by his party against his will D. The twins in Nigeria is _____. governor is being very serious with his second A. Mungo Pack term aspiration B. Mary Slessor C. Funmilayo Ransome Kuti 43. Moses has been booted out of his job by D. Queen Elizabeth his employer. A. Moses has been promoted by his employer 36. Regarding admission into Unilorin, I resign B. Moses has been commended by his myself to fate. employer A. I am prepared for whatever happens C. Moses has been sacked by his employer B. I am prepared to consult fortune teller D. Moses has been kicked by his employer C. I am prepared to contest the outcome D. I am prepared to resign my right of being Choose the best fit option that’s nearest admitted in meaning in the underlined words below. Use it to answer question 44-51. Instruction: Choose the correct interpretation to answer questions 37-43. 44. He has been inundated with work. A. Overwhelmed 37. The problem with fatal is that he has a B. Unhappy with finger in every pie. This means that _____. C. Interested in A. Fatai loves eating all kind of pies D. excited B. Fatai is involve in too many activities C. Fatai steal things 45.The vice chancellor has approved a new D. Fatai likes making pies allowance an incentive for the all lectures A. An advance 38. Peter's mother has kicked the bucket. This B. A package means that ____. C. An encouragement A. Peter grandmother is dead D. A reward B. Peter's grandmother is strong C. Peter grandmother is in the hospital 46. Although the speech was impromptu, it was very well. 39. I had a night out with Dickson. A. Prepared A. Dickson and I spent the night with reading B. Advertised B. Dickson and I had an evening of fun and C. Read out enjoyment D. Not prepared for C. Dickson and I quarrelled all night D. Dickson and I went outwww.prep.ng when it was night 47. My appeal to the lecturer for help was futile. 40. I am going for a work out this evening. A. Successful A. I am planning to Steal this evening B. Accepted B. I am going to visit a friend this evening C. Without success C. I'm going for physical exercise this evening D. Realistic D. I am going to my work this evening 48. Tonia was accused of immodest dressing 41. Michael took the English language exam A. Loud for granted. B. Poor A. He assumed the exam would be easy C. Indecent B. He did not worry about the out of the exam D. Costly C. He expected the exam would be difficult D. He worked hard for the exam 49. While preparing for the post jamb aptitude test, Felix read through pages 35 to 78 and

45

Download More FREE Past Questions at www.prep.ng

102 to 127 of an English text book. How many C. Sure pages did he read altogether? D. Uncertainty A. 58 B. 70 57. Choose the correct spelling of the word C. 68 below. D. 64 A. harasment B. harrasment 50. What happened last week at the meeting C. harassment vindicate my earlier position D. haressment A. Disagreed with B. Proved Instruction: Complete the idiom below C. Dismissed 58. A stitch in time ______. D. Voided A. revive all B. saves nine 51. Let me tell you, David is one student with C. saves all an impeccable character. D. revive nine A. Faultless B. Bad Instruction: Fill in the gap with the best C. Interesting fit in meaning. D. Deviant 59. John talks as if he ______everything. 52. The president visit the town with a view A. knew to_____ things for himself. B. known A. see C. knows B. been D. know C. seeing D. having seen 60. Choose the correct spelling of the word below. 53. Hide and Kemi quarrelled last week and A. acomodation haven't ____ yet. B. accommodation A. Made it out C. acommodation B. Made up D. accomodation C. Made out D. Made it up 61. If the ratio of girls to boys in a class is 5:3. Which of these cannot be the number of 54. It was _____ on the evening news. students in the class? A. Broadcasted A. 32 B. Being broadcasted B. 40 C. Been broadcast C. 48 D. Broadcast D. 36

Instruction: Choose the word that is 62. Who gave Nigeria her name and after what opposite in meaning inwww.prep.ng the underlined was Nigeria named ____ word(s) below. Use it to answer question A. Flora Michael and Niger Area 55-56. B. Lady Flora and Area of Niger C. Florence Shaw and Niger Era 55. Patience told the congregation that she D. Flora Shaw and Niger Area married a doting man A. A loving 63. Choose the correct spelling of the word B. A funny below. C. Uncaring man A. Quarrelling D. Nagging B. Quarelling C. Quarreling 56. He was very discreet about the number of D. Quareling university entrance examinations he has written. 64. The wicked boy threw a stone at the bird A. Secretive smashing _____ two legs. B. Open A. is 46

Download More FREE Past Questions at www.prep.ng

B. it's C. Extreme cold C. it D. Lack of weight D. its 73. What is the name of the white part of a 65. Which number comes next in the series 0, boiled egg? 1, 3, 6, 10, 15, ...... A. Yolk A. 26 B. Abdomen B. 21 C. Albumen C. 19 D. Egg white D. 32 74. The period in which the hen lies on its eggs 66. Ten litres of water _____ to be consumed until it aches is known as ______period. daily. A. Hibernation A. are B. Incubation B. is C. Breeding C. may D. Rousting D. should 75. How long does the chicken lie on its eggs 67. Choose the correct spelling of the word before it aches? below. A. 10 days A. Lackadasical B. 12 days B. Lackaidasical C. 30 days C. Lackaidaisical D. 21 days D. Lackadaisical 75. Which part of the egg of a chicken 68. The head of the Senate chamber of the develops into the young ones? National Assembly is _____. A. Yolk A. Secretary to the government B. Yellow matter B. Speaker C. White matter C. Chief whip D. Air space D. president 76. Animals that give birth to their young 69. After Jerry had made the bed, he ...... on ones alive are called____. it. A. Birds A. layed B. Reptiles B. lied C. Amphibians C. lay D. Mammals D. laid 77. What term is given to the changes insects 70. Who is the first Senate president in undergo in their life history? Nigeria? A. Recuperation A. Adolphus Wabara B. Rousting B. Ken Nnamani www.prep.ngC. Metamorphosis C. Nwafor Orizu D. None of the above D. Anyim Pius Anyim E. Nnamdi Azikiwe 78. What is the major product of the process of respiration in living things? 71. What structure in the fish serves the same A. Carbohydrate purpose as the lungs in human being? B. Protein A. Scales C. Energy B. Lateral line D. Carbon dioxide C. Fins D. Gills 79. Which of the following is a cereal? A. Maize 72. One of the major problems faced by the B. Cocoa men who travel into space is ___. C. Coffee A. Hunger D. Tea B. Lack of water 47

Download More FREE Past Questions at www.prep.ng

80. An organism that lives on a living organism and eventually causes harm to it is called ____. A. Herbivore B. Carnivore C. Parasite D. Pest

ANSWERS TO 2013 POST UTME QUESTIONS

1. C 2. B 3. C 4. C 5. D 6. D 7. D 8. C 9. A

10. D 11. D 12. C 13. A 14. C 15. B 16. A

17. C 18. A 19. D 20C 21. B 22. D 23. B

24. C 25. A 26. A 27 B 28. B 29. B 30. A

31. D 32. C 33. B 34. A 35. B 36. A 37. B

38. A 39. B 40. C 41. A 42. B 43. C 44. A

45. C 46. D 47. C 48. C 49. B 50. B 51. A

52. C 53. B 54. D 55. C 56. B 57. C 58. B

59. C 60. B 61. D 62. D 63. A 64. D 65. B

66. B 67. D 68. D 69. C 70. E 71. D 72. D

73. C 74. B 75. D 76. D 77. C 78. D 79. A

80. C

www.prep.ng

48

Download More FREE Past Questions at www.prep.ng

UNILORIN POST UTME PAST QUESTIONS 2014/2015

1. How many years did Nelson Mandela A. Tramelled spend as a political prisoner? B. Trammelled A. 37 C. Trammeled B. 11 D. Trameled C. 27 D. 7 10. Choose the correct spelling of the word below. 2. Who is the only African to win the world A. incesant best footballer of the year award? B. inccesant A. Samuel Eto'o C. incessant B. Didier Drogba D. inccessant C. George Best D. Abedi Pele 11. Benue state is known as the _____ of the nation. 3. Africa’s first female president is_____. A. Sunshine A. Sirleaf Ellen Johnson B. food basket B. Mrs Winnie Mandela C. coal state C. Mrs Ellen Borland D. harmony D. Mrs Rockland Mpholulu 12. The first man to win a Nobel laureate price 4. Choose the correct spelling of the word for literature in west Africa is? below. A. Chinua Achebe A. Etiquette B. Chike Obi B. Etiqquete C. Wole Soyinka C. Etiquete D. Chukwu Emeka D. Ettiquete 13. A lawyer is also referred to as a _____? 5. Choose the correct spelling of the word A. Legal expert below. B. Judge A. Hemorrhage C. Plaintiff B. Haemorhage D. Legal practitioner C. Hemorrhage D. Haemorrhage 14. The criminal is a son of no _____. A. Pedegree 6. How many geopolitical zones has Nigeria? B. Pedigree A. 5 C. Trace B. 4 D. Naughty woman C. 3 D. 6 15. /i:/ (Rhymes) A. leak 7. There are 20 houses inwww.prep.ng a community, 2 B. sickie houses were being sold. How many do we C. side have left? D. lick A. 18 B. 20 16. /u:/ (Rhymes) C. 16 A. book D. 17 B. look C. table 8. Our progress was ...... by the bad weather. D. school A. blocked B. posted 17. Aminat as well as her sisters _____ C. altered coming now. D. hampered A. are B. were 9. Choose the correct spelling of the word C. was below. D. is 49

Download More FREE Past Questions at www.prep.ng

C. over 18. Who designed Nigeria’s flag, what year, D. across where was he and what was he doing at that point in time? 26. He _____ arrived by now I can hear all the A. Ibrahim Taiwo, 1960, USA, Artist people shouting. B. Taiwo Hassan, 1958, Germany, Lecturer A. would have arrived C. Ibrahim Hassan, 1959, Australia, Designer B. must have arrived D. Taiwo Akinkumi, 1959, London, Student C. has arrived D. should have arrived Instruction: Fill in the gap with the best option. 27. Sibi ...... novels since she came home A. is reading 19. Peter's _____ a cold upset our plan. B. has read A. catches C. had been reading B. caught D. has been reading C. catching D. catch 28. Only observers from the ministry of works _____ allowed into the conference room. 20. How many colours has Nigeria’s flag A. is A. 3 B. are B. 1 C. have C. 4 D. has D. 2 29. Each of the candidates that came late ____ 21. Birds of _____ flocks together. to complete _____. A. some feathers A. have/this form B. the same feathers B. are having/these forms C. a feather C. has/this form D. like feathers D. have/these forms

1 2 22. Arrange these in ascending order /2 , /3, 30. The board has been having _____ series of 3 4 /4 and /5 meetings lately. 1 2 3 4 A. /2, /3, /4, /5 A. a 2 1 4 3 B. /3, /2, /5, /4 B. some 4 3 2 1 C. /5, /4, /3, /2 C. a lot of 2 3 4 1 D. /3, /4, /5, /2 D. many

23. The city which was once called the 31. The patient is suffering _____. “forbidden city” is in? A. Reumatism A. Tokyo, Japan B. Rheumantism B. Berlin, Germany C. Rhuematism C. Peking, China D. Rheumatism D. Calabar, Nigeria. www.prep.ng 32. The attitude of my students to _____ For questions 24-34, choose the option baffles me. that best completes the gap(s) A. Pronunciation B. Pronounciation 24. Abuja officially became Nigeria's capital on C. Prononciation ____? D. Pronuonciation A. 1st January 1992 B. 5th May 1991 33. By the time the plane reaches Harare, it C. 1st October 1992 ____ in the air for twelve hours. D. 12th December 1991 A. has been B. should be 25. I ran ____ an old friend of mine on broad C. would have been street and brought him home. D. would be A. into B. to 34. Those _____ are very beautiful. 50

Download More FREE Past Questions at www.prep.ng

A. flowers of her C. MOtivation B. flowers of her's D. motivaTION C. our flowers D. flowers of ours 42. certification A. cerificaTION 35. My mother bought a BICYCLE yesterday. B. certifiCAtion A. what did my mother buy yesterday? C. cerTIficafion B. whose mother bought a bicycle yesterday? D. certiFIcation C. did my mother steal a bicycle yesterday? D. when did my mother buy a bicycle? 43. programmatic A. PROgrammatic 36. Musa is STAYING in Enugu. B. prograMAtic A. Is Musa staying on the outskirts of Enugu? C. programmatiC B. Is Audi staying in Enugu? D. programmatic C. Was Musa staying in Enugu? D. Is Musa passing through Enugu? For questions 44-48, choose the option that best completes the gap(s) Instruction: Choose the option that has the same vowel sound as the one 44. Oboro will always _____ his friends. represented by the letter(s) underlined. A. stand up for B. stand down for 37. epitaph C. stand across for A. pseudo D. stand beside for B. paper C. pneumonia 45. Yours is to command, _____ is to obey. D. fan A. their's B. theirs' For questions 38-39, choose the option C. theirs that best completes the gap(s) D. their

38. The woman refused to testify ____ her 46. Local government are authorised to pass husband. _____. A. in A. byes'-laws B. at B. bye-laws C. from C. bye-law D. against D. bye's-law

39. The guard spent all night pacing_____. 47. The philanthropist devoted himself _____ A. from and to the poor. B. fro and to A. by helping C. to and from B. in helping D. to and fro C. to helping www.prep.ngD. at helping 40. He went to the restaurant to enjoy the special _____. 48. Umar: I have never visited the dentist A. suite Aliyu: .... B. a la carte A. neither I C. chef B. neither myself D. cuisine C. neither have I D. I also never For these questions, choose the appropriate stress pattern from the Instructions: For these questions, choose options, the stress syllables are written in the nearest in meaning to the word capital letter(s) underlined.

41. motivation 49. Emeka's painting was so realistic that it A. moTIvation could almost have been a photograph B. motiVation A. picturesque 51

Download More FREE Past Questions at www.prep.ng

B. concrete Instruction: From question 57-59, choose the C. lively option that best completes the gap(s). D. authentic 57. My father has been away on a journey but 50. Courteously, Ade stood back to let his my mother says she ____ he ____this Friday. teacher go first through the door. A. expects/would A. patiently B. expected/will arrive B. politely C. expects/will arrive C. carefully D. expected/would arrive D. calmly 58. Kola was fined; _____thieves were given a For question 51, choose the option that jail sentence. best completes the gap(s) A. rest B. the rest of the 51. A nursery rhyme is used to teach pupils C. the rest how to spell the word _____. D. rest of the A. hipopotamus B. hippoppotemus 59. Do you mind ____ another hour or two? C. hippopotamus A. wait D. hippoppotemus B. waiting C. to have waited For question 52-53, choose the D. to wait appropriate stress pattern from the options, the stress syllables are written in 60. Round off 1.56 to 1 decimal place. capital letter(s) A. 1.6 B. 1.56 52. jurisprudence C. 0.56 A. JUrisprudence D. 1.5 B. juRISprudence C. jurisPRUdence 61. Simplify 7+y = 20. Find y D. jurispruDENCE A. 18 B. 27 53. suburbanite C. 13 A. SUburbanite D. 14 B. suBURbanite C. suburBAnite 62. Simplify 10 + p = 30. Find p D. suburbaNITE A. 30 B. 10 C. 20 54. The EAGLES won the match D. 40 A. Did the eagles lose the match? B. who won the match? 63. What is the total charge for a distance of C. what did the eagle win?www.prep.ng 4km if a taxi charges 50k per kilometre. D. Did the eagles win the match? A. ₦ 5.00 B. ₦ 2.00 55. For question 55-56, Choose the correct C. ₦ 9.00 spelling below. D. ₦ 4.00 A. comittee B. commitee 64. Scientists who study the weather are called C. comittee _____. D. committee A. Geographers B. Archaeologist 56. A. Catarrrh C. Astrologers B. Catar D. Meteorologist C. Catarr D. Catarrh 65. A reflection in a mirror is called _____. A. Darkness B. Image 52

Download More FREE Past Questions at www.prep.ng

C. Shadow D. Colour 73. How long does a train take to cover the journey if it starts at 6.10a.m. and ends at 66. On a map, two meters represent fifteen 3.55p.m. the same day? kilometres. What length on the map would A. 10hrs 35mins represent a distance of ninety kilometres? B. 9hrs. 45mins A. 675 meters C. 9hrs 30mins B. 337.5 meters D. 10hrs 45mins C. 12 meters D. 9 meters 74. A clock gains 20 seconds every hour. It shows the correct time at 6 a.m. What time 67. If ₦1200 is divided between boys A, B and will it show at 3p.m.? C at a ratio of 5:2:1 respectively, what is the A. 3.03p.m. share of boy B? B. 3.06p.m. A. ₦600 C. 2.57p.m. B. ₦750 D. 3.13p.m. C. ₦300 D. ₦150 75. Find the number of days from 12th June E. ₦400 noon to 12th September noon. A. 90days 68. Dotun, Ayo, Stephen and Olu were given B. 92days 10, 18, 24 and 8 oranges respectively. What C. 61days percentage of the total oranges was given to D. 62days Stephen? A. 40 76. A plane leaves town B at 09.50 hours and B. 18 arrives town C at 19.40hours. If the time at C C. 25 is one hour ahead of time B how long does the D. 7 flight take? E. 30 A.8hrs 45mins B. 8hrs 50mins 69. What is the speed of a car that travelled C. 7hrs 55mins 840km in 7 hours? D. 7hrs 45mins A. 7km/hr B. 120km/hr 77. Find the HCF of 36 and 60 C. 940km/hr A. 3 D. 110km/hr B. 6 E. 130km/hr C. 12 D. 36 70. The judge ______her to two years 16 imprisonment 78. Reduce /36 to its lowest term. 16 A. pronounced A. /36 8 B. commended B. /18 4 C. condemned www.prep.ngC. /9 2 D. tried D. /3 E. sentenced 3 79. Express /4 in a decimal form. 71.Scurvy is a disease caused by lack of which A. 0.075 vitamins? B. 0.75 A. A C. 7.50 B. B D. 0075 C. C D. E 80. A teacher earns ₦15,000 a month. He spends ₦9000 and saves the remaining 72. Collection of stars is called ______. amount. What is the ratio of his savings to the A. Stardom amount he spends? B. Galaxy A. 3:2 C. Astronomy B. 2:3 D. Meteorology C. 3:5 53

Download More FREE Past Questions at www.prep.ng

D. 5:3

SOLUTIONS TO POST UTME 2014

1. C 2. C 3. A 4. A 5. D 6. D 7. B 8. D 9. B

10. C 11. B 12. C 13. D 14. A 15. A 16. D

17. D 18. D 19. C 20. D 21. C 22. A 23. C

24. D 25. A 26. B 27. D 28. B 29. C 30. A

31. D 32. A 33. C 34. D 35. A 36. D 38. D

39. D 40. D 41. B 42. B 43. D 44. A 45. C

46. B 47. C 48. C 49. A 50. B 51. C 52. C

53. B 54. B 55. D 56. D 57. C 58. B 59. B

60. A 61. C 62. C 63. B 64. D 65. B 66. C

67. C 68. A 69. B 70. E 71. C 72. B 73. A

74. A 75. B 77. C 78. C 79. B 80. B

www.prep.ng

54

Download More FREE Past Questions at www.prep.ng

UNILORIN POST UTME PAST QUESTIONS 2015/2016

1. The largest known planet is______. A.5% A. Sun B. 2 ½ % B. Uranus C. 6% C. Jupiter C. 4% D. Earth 9.The daily sales in a week at a petrol station 2. The largest continent on earth is ____? are 100 litres, 825 litres, 707 litres, 830 litres, A. Africa 642 litres, 908 litres and 112 likes. What is the B. Australia average daily sales? C. North America A.848 litres D. Asia B. 589 3/7 litres E. Europe C. 718 litres D. 617 litres 3. 50 men can build a house in 60 days. How many more men of equal strength and ability 10. A car traveling at 80 km per hour leaves must be put on so as to finish a similar house Lagos at 8.am for Ibadan 136km away. What in 40 days? time did it get to Ibadan? A. 20 men A.8.42am B. 30 men B. 9.42am C. 35 men C. 9.32am D. 25 men D. 9.30am E. 45 men 11. By how much is 845.79 greater than 4. A group of letters that is added to the 99.359? beginning of a word in order to change its A. 746.431 meaning e.g ”un’’, “anti’’, is ______? B. 74.6431 A. Suffix C. 835.8541 B. Antonym D. 83.5841 C. Prefix D. Pronouns 12. A square lawn has an area of 729 sq.m. What is the distance around the lawn? 5. Pastoral nomadic is a common characteristic A. 96 of the ______. B. 108 A. Hausas C. 120 B. Igbos D. 84 C. Fulanis D. Yorubas 13. In a school, the ratio of girls to boys is 5:6. E. Edos How many are boys if the total population is 605? 6.What fraction of ₦4.32 is ₦3.87 A. 365 41 A. /48 www.prep.ngB. 275 43 B. /48 C. 330 47 C. /48 D. 560 39 D. /48 14. A class-one teacher gave 23 counting 7. Yinka and Segun have 120 oranges Yinka seeds to each of the 45 pupils in his class. How gets 16 oranges more than Segun. How many many counting seeds did the teacher give out? oranges has Segun? A. 225 seeds A. 60 B. 935 seeds B. 26 C. 2070 seeds C. 68 D. 1035 seeds D. 52 15. Find the greater number that will divide 8. If 480.00 borrowed for 3 years simple 43, 91 and 183 so as to leave the same interest became ₦516, what was its rate of remainder in each case. interest? A. 4 55

Download More FREE Past Questions at www.prep.ng

B. 7 C. ₦56.00 C. 9 D. ₦44.00 D. 13 24. What principal will gain ₦168.00 simple 16. The total number of digits used in interest in 4 years at 7½ % numbering the pages of a book having 336 A. ₦280.00 pages is: B. ₦480.00 A. 732 C. ₦560.00 B. 990 D. ₦720.00 C. 1098 D. 1305 25. Which country won the Olympic gold medal for football during 1996 Atlanta Olympic? 17. Which of the following pairs of A. USA fractions adds up to a number greater than 5? B. France 5 3 A. /3, /4 C. Nigeria 7 11 B. /3, /5 D. Brazil 11 8 C. /4, /3 13 11 D. /5, /6 26. Which of the following is not a tree crop? A. Yam 18. What is the difference between the biggest B. Cashew 2 3 4 and the smallest fraction among /3, /4 , /5 C. Cocoa 5 and /6? D. Coffee 1 A. /6 1 B. /12 27. Which of these animals does not have 1 C. /20 horns? 1 D. /30 A. Goat B. Cattle 19. What number whose ½ is multiplied by 1/3 C. Buffalo of the same number will give a product of 726? D. Horse A.96 E. Ram B.166 C.160 28. Which country boarders Nigeria in the D. 66 west? A. Ghana 20. What is the product of 0.101 and 11.1? B. Republic of Benin A. 12.21 C. Togo B. 1.1211 D. Cameroun C. 1.211 E. Chad D. 0.11211 29. A tripod has how many supports? 21. What is 7/8 of the profit of a contractor in A. 2 a year, if half of the profits made is ₦2880.72? B. 4 A. ₦5040.26 www.prep.ngC. 3 B. ₦5041.26 D. 5 C. ₦2520.63 E. 6 D. ₦5041.36 30. Tyres are made from? 22.When 7867 is divided by 14, what remains? A. Pulp A. 11 B. Steel B. 13 C. Cotton C. 3 D. Rubber D. 7 E. Sizer

23. Eggs now sell at ₦2.55 a dozen and 31. The position of planet earth from the sun oranges 2 for 10k. What will 20 dozen eggs in the solar system is ______. and 100 oranges cost? A. 3rd A. ₦48.00 B. 2nd B. ₦60.50 C. 1st 56

Download More FREE Past Questions at www.prep.ng

D. 4th E. 5th 40. There is a big hole in my pocket means ______. 32. The name of a young lion is called _____. A. my pocket is leaking A. kitten B. my money comes and goes away fast. B. kevlin C. my pocket is torn C. cub D. my pocket is not in good shape D. cude 41. He got the job and suddenly became 33. The name of an equipment for measuring swollen headed pressure is called ____. A. The job is full of hazards A. barometer B. The job made him become proud. B. thermometer C. the job made him a big man C. gauge D. The job made him wealthy D. hygrometer 42. Which of the following is arranged in wrong For question 34-40, Choose the word that best ascending order? completes the gap(s). A. Feet-Body -Head 34. It is right to say, switch _____or put_____ B. Middle-Top-Bottom the light subject to the type of light. C. Root-Stem-Leaves A. of/off D. Sea-Land-Mountain B. off/off C. out/off 43. Which correspondence is wrong matching? D. off/out A. nose-breathing-air B. mouth-laughing-sound 35. A betrayal from someone supposed to be a C. eyes-seeing-heat friend is a stab ______the back D. feet-motion-distance A. On B. In 44. Which of the following is the correct term C. At for entering an aircraft? D. From A. enters B. board 36.The handset won’t work; the battery has C. transit run______. D. tranced A. Off B. Out 45. Which of the following is wrong? C. Down A. Plane crash D. Over B. boat sink C. Car accident 37. The thief was caught. He couldn’t get D. ship wreck _____with what he stole. A. Away 46. The officials accompanying a VIP for B. Scot free www.prep.ngprotection purpose are referred to as C. Free A. Police D. Over B. Escort C. Soldier 38. The University project ______owing to D. Mobile lack of funds. A. Fell down 47. Someone who persist in doing the wrong B. Fell over thing even after he has been punished is said C. Fell off to be D. Fell through A. stubborn B. heady 39. “He must carry his cross” means ____. C. recalcitrant A. She is a Christian D. disobedient B. He is a disciple C. He is a minister of God 48. In which of the following African countries D. He suffers by destiny will you find a confluence of two oceans. 57

Download More FREE Past Questions at www.prep.ng

A. South Africa A. A mathematical complex inferior to B. Nigeria algorithm. C. Somalia B. A feeling that one is not as good or D. Senegal intelligent as other people. C. A mathematical complex describing an 49. Choose the odd one out among the inferior integral followings: D. A life inferior complex A. Peru B. Venezuela 58. The term LAN in Computer Systems means C. Bolivia ____. D. Indonesia A. Local Area Network B. Low Antenna Network 50.The capital city of Rwanda is _____. C. Low Attenuated Network A. Khartoum D. logarithmic Advanced Network B. Kabul C. Kigali 59. A ______is one of the tools for bricklaying. D. Kingston A. Spanner B. Nail 51. The geographic equator passes through C. Trowel which of the following countries? D. Scriber A. Uganda B. Zambia 60. The following maintains law and order C. Nigeria EXCEPT. D. Cameroon A. Police B. Air force 52. Which of the followings is not a planet? C. Engineer A. Pluto D. Civil Defence B. Mercury C. Venus 61. Good reading culture is done in D. Moon the...... A. Darkroom 53. Which of the followings is a star? B. Bathroom A. Earth C. Library B. Sun D. Kitchen C. Moon D. Nebula 62. Nigeria does not have this military branch the United States of America has _____. 54. The River Niger has its source from _____. A. Navy A. Fouta Djallon Island B. Air force B. Lake Guinea C. Infantry C. Upper Volta D. Marine D. Timbuktu www.prep.ng63.In which state is Madonna University 55.The earth is mainly covered with _____. situated in Nigeria? A. Water A. Kwara B. Land B. Ekiti C. Equal amount of land and water C. Anambra D. Mountains D. Bayelsa

56. FM in radio operation means _____. 64. When did Nigeria join the World Trade A. Far medium transmission Organisation? B. Mega Frequency transmission A. 1995 C. Modulated Frequency transmission B. 1904 D. Modern Frequency transmission C. 2004 D. 1996 57. Inferiority complex is used to describe ____. 65. The following are stationery EXCEPT _____. 58

Download More FREE Past Questions at www.prep.ng

A. Stapler B. Exercise book 73. Seconds, minutes, hours are units with C. Abacus definite converts faction which of the following D. Office pin is not _____. A. inch, feet yard 66. The following are types of camera B. naira, dollar, pound EXCEPT. C. centimetre, meter, kilometre A. Kodak D. centigrade, Fahrenheit B. Samsung C. Sony D. Supper Kodak 74. Mrs. Felicia Ojo borrowed ten (10) tiers of garri from her neighbour, Mrs. Comfort 67. The full meaning of WAEC is _____. Chinedu and promises to return it three-fold, A. West African Examinations Council the following month. How many tiers of garri B. West Africa Examination Council will she return? C. West Africa Examination Councils A. 3 D. West Africa Examination Council B. 13 C. 30 Instruction: Choose a word that is D. 10 opposite in meaning to the first word. 75. Divide 666 cowries between 6 women 68. Spendthrift: traditional gods worshipers. How much does A. Miser each receive? B. Loan A.111 C. Foolish B. 110 D. Happy C. 66 D. 660 Instruction: Choose a word that is opposite in meaning to the first word. 76.Write in full figures 106. A. 60 69. Profane: B. 6.0 A. Beautiful C. 106 B. Sacred D. 1,000,000 C. Profuse D. Stiff. 77. I am 10 years old my sister is 4, in how many years shall I be twice as old as she will 70. A team of eight lumberjacks cut an be? average of 15,000 cubic feet of timber in a A. 3 week. How many cubic feet will four B. 4 lumberjacks cut in four weeks? C. 2 A. 30, 000 D. 5 B. 25,000 C. 32,000 www.prep.ng78. Bola had fewer sums right than Ojo, Ojo D. 16,000 had fewer sums right than Dele. Which had the most right answers? 71. The expression 2 x 2 x 2 is equal to 8 can A. Bola be expressed as ____. B. Ojo and Bola A. quadruple 2 C. Dele B. triple 2 D. Bola and Dele C. cubic 2 D. thrice 2 79. Audu is taller than Laide but shorter than Deji. Deji is of the same height as Taiwo.Who 72. The head of a parliamentary system of is the shortest? Government is _____. A. Audu A. The Prime Minister B. Laide B. The President C. Deji C. The Queen D. Taiwo D. The King 59

Download More FREE Past Questions at www.prep.ng

80. The first prize was ______tray. A. a carved wooden attractive B. a wooden attractive carved C. an attractive carved wooden D. a carved attractive wooden

ANSWERS TO UNILORIN 2015 POST UTME

1. C 2. D 3. D 4. C 5. C 6. B 7. D 8. B 9. B

10. B 11. A 12. A 13. C 14. D 15. A 16. B

17. C 18. A 19. D 20. B 21. B 22. B 23. C

24. C 25. C 26. A 27. D 28. B 29. C 30. D

31. A 32. C 33. A 34. D 35. B 36. C 37. A

38. D 39. D 40. B 41. B 42. B 43. C 44. B

45. B 46. B 47. C 48. A 49. D 50. C 51. A

52. D 53. B 54. A 55. A 56. C 57. B 58. A

59. C 60. C 61. C 62. D 63. C 64. A 65. C

66. D 67. A 68. A 69. A 70. A 71. C 72. A

73. B 74. C 75. A 76. D 77. C 78. C 79. B

80. C

**NO POST-UTME was conducted in 2016

www.prep.ng

60

Download More FREE Past Questions at www.prep.ng

UNILORIN POST UTME PAST QUESTIONS 2017/2018

1. Who wrote the novel “Half of a yellow sun”? 10. The Federal Capital Territory was shifted A. Chinua Achebe from Lagos to Abuja in what year? B. Wole Soyinka A. 1989 C. Olusegun Obasanjo B. 1990 D. Chimamanda Adichie C. 1991 D. 1992 2. Who is the author of Things Fall Apart? A. Wole Soyinka 11.Who designed the National flag? B. Chimamanda Adichie A. Mr. Taiwo Akinkunmi C. Chinua Achebe B. Mr. Taiwo Akinyemi D. Flora Nwapa C. Mr. Taiye Akinwunmi D. None of the above 3. Which of the following spellings is correct A. Panthar 12. Nigeria became a republic in what year? B. Panther A. 1960 C. Pantha B. 1914 D. Panthre C. 1980 D. 1963 4. The old woman ambles about in her garden A. moves hurriedly 13. In what year did the Northern and B. moves noisily Southern protectorate in Nigeria amalgamate? C. moves slowly A. 1960 D. moves fast B. 1963 C. 1914 5. Which of the following is correctly spelt? D. 1991 A. brocolli B. broccolli 14. Which of these is correctly spelt? C. brocollil A. Millennium D. broccoli B. Millenium C. Milinneum 6.In a single throw of a dice, what is the D. Millinneum probability of getting a number greater than 4? 1 A. /2 15. Muhammadu Buhari became the military 2 B. /3 head of states in Nigeria in what year? 1 C. /4 A. 1985 1 D. /3 B. 1984 C. 1983 7. In banking, ATM stands for ____? D. 1982 A. Automated Tallying Machine B. Automated Teller Machine 16. Which of these is correctly spelt? C. Automated Totaling Machinewww.prep.ng A. Colleague D. Automated Transaction of Money B. Collaegue C. Colleage 8. Late Idi Amin Dada was from which of the D. coleague following countries A. Kenya 17. Olusegun Obasanjo ruled as a Military head B. Tanzania of states from what year to what year? C. Uganda A. 1986 - 1989 D. Congo Brazzaville B. 1976 - 1979 C. 1975 - 1978 9. The following Nigerian footballers have won D. 1999 - 2007 the African Footballer Award except A. Victor Ikpeba 18. The present senate of Nigeria is what B. Nwankwo Kanu republic? C. Rasheed Yekini A. Eighth D. Austin Okocha B. Seventh 61

Download More FREE Past Questions at www.prep.ng

C. Sixth B. laudable D. Fifth C. uninspiring D. cautious 19. Which of the following is correct? A. Laison 26. The company is to shed three thousand B. Liaison staff this year. C. Liason A. demotes D. Laisson B. lay off C. throw up 20. Apparition is synonymous to D. placates A. Ghost B. Spirit 27. There was a glut of oil on the market. C. Demon A. a variety of D. Witch B. an accumulation of C. an abundance of 21. Ngozi has always considered her father to D. an increase in be an impassioned man. A. Her father is a very lively man. 28. The nurse was in favour of voluntary B. Her father is an emotional man. euthanasia. C. Her father is a disciplined man. A. a painless death D. Her father is a very strict man. B. a simple operation C. a sleeping pill 22. The manager paid us in hard currency. D. a major operation A. We were paid in new notes. B. We were paid in foreign currency. In each of question 29-30: choose the C. We were paid in dollars and pound sterling. option word that best completes the D. We were paid in a strong and stable gap(s) currency 29. Be careful not to _____ this money. 23. If he went to London, he would see the A. lose Queen. B. loose A. When he goes to London, he will see the C. loss Queen. D. lost B. He did not go to London and did not see the Queen. 30. Ali plays _____ violin with remarkable skill. C. He did not see the Queen when he went to A. their London. B. some D. He would like to see the Queen when he C. a goes to London. D. the

24. The elders rebuked Olu for taking issue with his principal. For question 31-33, choose the option A. Olu was cautioned for www.prep.ngshouting at his that is nearest in meaning to the word principal. underlined. B. Olu was scolded for acting in collusion with his principal. 31. Ugo has often been described as C. Olu was reprimanded for arguing with his belligerent principal. A. attractive D. Olu was blamed for issuing a statement B. patient denying his principal. C. innocent D. combative For question 25-27: choose the option that is nearest in meaning to the word 32. The player kept on gamely to the end of underlined. the match A. amateurishly 25. The leader has the unstinting support of B. skillfully his party. C. courageously A. unsparing D. stubbornly 62

Download More FREE Past Questions at www.prep.ng

Choose the option word that best 33. Tade became timorous when she was completes the gap(s) asked to give the valedictory speech A. excited 40. A wide range of options ...... made B. nervous available to the political parties during the C. aggressive recently concluded elections. D. happy A. are B. were Instruction: For question 34-35, The word in C. was capital letter has the emphatic stress. D. is Choose the option to which of the given sentence relates. For question 41-42, Choose the option 34. YOUR sister should come with us that has the same consonant sound as the tomorrow. one represented by the letter(s) A. Should your brother come with us tomorrow underlined. B. Should our brother come with us tomorrow? C. Should Ado's sister come with us tomorrow? 41. Past D. Should my sister come with us tomorrow? A. wrestle B. preached 35. They FLEW to Abuja C. castle A. Did they go to Abuja by road? D. pasture B. Did they fly to Jos? C. How will they get to Abuja? 42. Sure D. Where did they fly to? A. cheer B. cheap In each of question 36-38 and 40: choose C. charlatan the option word that best completes the D. church gap(s) Choose the appropriate stress pattern 36. The imposing edifice ______a fortune to from the options, the stress syllables are build. written in capital letter(s) A. had costed B. have cost 43. Expostulate C. costed A. exposTUlate D. cost B. expostuLATE C. EXpostulate 37. When Ajike met her ______husband at the D. exPOStulate party she felt like reconciling with him. A. estranged For question 44-49, Choose the option B. caring word that best completes the gap(s) C. strange D. loving 44. The train ______before I arrived. www.prep.ngA. was leaving 38. Three quarters of the hostel ______been B. was leaving painted and three quarters of the students C. had left _____ moved in. D. would leave. A. has/has B. has/have 45. ______of the students turned up, so the C. have/has lecture was cancelled D. have/have A. few B. little 39. Church C. a few A. feature D. a little B. chauffeur C. Ocean 46. You don’t like mathematics, _____you? D. Machine A. don’t B. do C. can’t 63

Download More FREE Past Questions at www.prep.ng

D. can B. 3:4 C. 1:3 47. If you are not careful, you would ____ D. 5:12 your money. 55. I have ₦12 and my brother has ₦24. what A. loss is the ratio of my money to my brothers? B. lose A. 3:4 C. loose B. 1:2 D. lost C. 2:4 D. 1:3 48. He ____ the picture on the table. A. lay 56. A decagon has ______sides B. laid A. Seven C. lied B. eight D. lain C. nine D. ten 49. By the end of this month, ______for three years in this school. 57. A dozen is 12, a score is ______A. I will study A. 20 B. I am studying B. 24 C. I will have been studying C. 48 D. I had been studying D. 144

Choose the appropriate stress pattern 58. What year was the University of Ilorin from the options, the stress syllables are established? written in capital letter(s) A. 1985 B. 1975 50. Photography C. 1974 A. phoTOgraphy D. 1984 B. PHOtography C. photoGRAphy 59. Who is the current Jamb registrar? D. photograPHY A. Prof ‘Dibu Ojerinde B. Prof Ishaq Oloyede 51. The rent for a house is ₦600 per month. C. Prof Itsey Sagay What is the total rent for a year? D. Prof Yemi Osibanjo A. ₦2720 B. ₦7200 60. Which of these is Kwara State slogan? C. ₦1720 A. God's own state D. ₦720 B. Food basket of the nation C. The big heart 52 Simplify (√49)2 D. State of harmony A. 7 B. 49 www.prep.ng61. If twenty five percent of candidate for post C. 6 UTME are girls, what fraction of the candidate D. 8 are boys? A. one - quarter 53. A lucky draw price of ₦150 was shared B. two - third among three winners in the ratio 4:5:6. How C. half much did each get? D. three – quarter A. ₦30, ₦40, ₦70 B. ₦40, ₦50, ₦60 62. Two sums of money are in ratio two to ten, C. ₦50, ₦60, ₦40 if the smallest is fifteen kobo, what is the D. ₦20, ₦50, ₦70 largest? A. one naira fifty kobo 54. In a school, 25 teachers out of 60 own B. thirty-five kobo cars. What is the ratio of the teachers who C. seventy-five kobo own cars to those who do not? D. one hundred and fifty naira A. 5:7 64

Download More FREE Past Questions at www.prep.ng

63. When Caleb was 15 years old his brother C. fahreheit Clement was 18years, if Caleb was born in D. fahrrenheit year 1900, in what year was Clement born? A. 1903 71. Which of this is correct? B. 1915 A. Rhinoceros C. 1897 B. Rinoceros D. 1867 C. Rhinocerous D. Rhinoxerous 64. In a class of thirty-two boys, sixteen passed an examination and four were absent 72. There is no love lost between them. while the rest students failed. What percentage A. they can’t do without each other. of the whole class failed? B. they are happy together. A. 37.5 C. they despise each other. B. 50 D. they are friends with each other. C. 42.86 D. 57.14 73. Which of this is the correct spelling. A. servette 65. I owe a sum of two hundred and twenty B. serviette naira and was able to pay one hundred and ten C. serviete naira. What percentage have I not paid? D. servitte A. forty – four B. twenty 74. NCC means _____. C. fifty A. Nitel Communication Commission D. fifty – four B. Nigerian Communication Centre C. Nigeria Communication Centre 66. He was both a writer and a politician, but D. Nigerian Communication Commission he was better _____a singer. A. as if 75. Which is the world’s largest archipelago? B. like A. British Isles C. as B. Canadian Arctic Archipelago D. to be C. Japanese Archipelago D. Malay Archipelago 67. We visited his house ...... three times. A. like 76.Where is British Isles located? B. for like A. Atlantic Ocean C. about B. Arctic Ocean D. for about C. Indian Ocean D. Pacific Ocean 68. We stood up when the principal came in, ...... 77. Where is the RING OF FIRE located? A. Isn’t it? A. Baltic Sea B. Didn’t we? B. Dead Sea C. Not so? www.prep.ngC. Indian Ocean D. Did us? D. Pacific Ocean

Choose the appropriate stress pattern 78.The two colours seen at the upper part of from the options, the stress syllables are the rainbow are ____. written in capital letter(s) A. Indigo and Violet B. Red and Orange 69. demarcation C. Green and Blue A. demarcaTION D. Orange and Yellow B. demarCAtion C. deMARcation 79.Which of these numbers is the odd one out? D. DEmarcation A. 4561 B. 3724 70. Which of this is correct? C. 9142 A. Fahrenheit D. 6434 B. farenheit 65

Download More FREE Past Questions at www.prep.ng

80.The cost of 20 pieces of tomato each weighing 150g is ₦240, what will be the cost of 30 pieces if each weighs 120g? A. ₦240 B. ₦300 C. ₦288 D. ₦388

SOLUTIONS TO 2017 PUTME QUESTIONS

1. D 2. C 3. B 4. C 5. D 6. D 7. D 7. B 8. C

9. D 10. C 11. A 12. D 13. C 14. A 15. C

16. A 17. B 18. A 19. B 20. A 21. B 22. D

23. B 24. C 25. A 26. B 27. C 28. A 29. A

30. D 31. D 32. C 33. B 34. D 35. A 36. D

37. A 38. B 39. A 40. C 41. B 42. C 43. D

44. C 45. A 46. B 47. B 48. B 49. C 50. A

51. B 52. B 53. B 54. A 55. B 56. D 57. A

58. B 59. B 60.D 61. D 62. C 63. C 64. A

65. C 66. C 67. C 68. B 69. B 70. A 71. A

72. C 73. B 74. D 75. D 76. A 77. D 78. B

79. D 80. C

www.prep.ng

66

Download More FREE Past Questions at www.prep.ng UNILORIN POST UTME PAST QUESTIONS 2018/2019

1. Jide and Mary met ___the University of 10. What is the sum of the smallest and the Ilorin carnival night. largest prime number from 1-10? A. At A. 8 B. In B. 9 C. On C. 10 D. From D. 11

2. Have you any objection ____lunch. 11. Which one of the following was NOT an A. To take American president? B. Taking A. Nelson Mandela C. To taking B. Barack Obama D. For taking C. Ronald Reagan D. George Bush 3. How many senate seats do we have in each state in Nigeria. 12. Choose the option that rhymes with the A. 3 word below. B. 4 Table C. 5 A. Tablet D. 2 B. Scramble C. Label 4. Who's the founder of Facebook? D. Sabre A. Bill Gate B. Mark Zuckerberg 13. As far as Ade is concerned, Kemi should C. Jimmy Carter not be given more than ₦50 at most. This D. Mike Adenuga means that ___. A. By Ade's estimation, Kemi doesn't merit 5. Those toys were ...... by him. ₦50 A. Made B. According to Ade, Kemi should be given not B. Make less than ₦50 C. Did C. All Ade is concerned with is that Kemi D. Done should not be given more than ₦50 D. Ade said that Kemi must be given ₦50 6. What's the work of lumberjack? A. Felling tree 14. Choose the option that has the same B. Planting tree sound as the word below Choice C. Planting flowers A. Chose D. Planting vegetable B. Boys C. Lounge y 10 7. If /5 = /25, Find y. D. Eyes A.1 www.prep.ng B. 2 15. If 3(x - y)=27 and 3(x + y)=243, find x C. 3 A. 1 D. 4 B. 2 C. 3 8. What is the capital of Gambia? D. 4 A. Banjul B. Bangui 16. What's the meaning of period? C. Monrovia A. clock D. Ndjamena B. semi colon C. comma 9. Budapest is in which country? D. full stop A. Netherlands B. Hungary 17. Verbs that remain the same in the past C. Argentina and past participle form are called? D. Portugal A. Regular verbs 67

Download More FREE Past Questions at www.prep.ng

B. Irregular verbs B. Prof C. Static verbs C. Prof Yakubu Mahmoud D. Dynamic verbs D. Prof Amina Zakari

18. Which of this ethnic groups is NOT 27. In 'bibliophobia' what does "biblio" stands Nigerian? for? A. Jukuns A. Disease B. Yorubas B. Bible C. Mendes C. Book D. Idomas D. Blue

19. They arrested the policemen that____the 28. How many syllables are there in the word women. (Prompt, Twelfth, Cart)? A. Harased A. 2 B. Harrassed B. 3 C. Harrased C. 4 D. Harassed D. 1

20. I'll do it in my___. 29. Which of these is associated with clusters? A. Leisure A. Diphthongs B. Leizure B. Monophthongs C. Lesure C. Consonants D. Liesure D. Vowels

21. Chose the correct spelling 30. How many players make up a volley ball A. Demacation team? B. Demercation A. 6 C. Dermacation B. 5 D. Demarcation C. 7 D. 8 22. Who is regarded as the king of pop? A. Tupac Shakur 31. Which of these pairs is consonantal? B. King Sunny Ade A. /P/ and /S/ C Michael Jackson B. /I/ and /E/ D. Ayo "Wizkid" Balogun C. /O/ and /P/ D. /A/ and /E/ 23. Athens is in which country? A. Greece 32. What is 40% of 50? B. Romania A. 10 C. France B. 15 D. Armenia C. 20 D. 30 24. Kalahari Desert is locatedwww.prep.ng in which part of Africa? 33. Which country has never won a World A. North Cup? B. West A. England C. East B. Uruguay D. South C. USA D. Brazil 25. Which country in the world has the has largest economy? 34. Which of these is the most commonly used A. China search engine? B. USA A. Yahoo C. Nigeria B. Ask.com D. Japan C. Google D. Facebook 26. Who is the current INEC chairman? A Prof 35. It is Ade and you, who ___wrong. 68

Download More FREE Past Questions at www.prep.ng

A. Is B. 1960 B. Are C. 1930 C. Am D. 1928 D. Was 45. The acronyms EFCC stands for? 36. Northern and Southern Nigeria was A. Economic and Finance Crime Commission amalgamated in what year? B. Economical and Financial Crime Commission A. 1919 C. Economic and Financial Crime Commission B. 1914 D. Economic and Financial Commission for C. 1960 Crime D. 1963 46. What is the full meaning of NSCDC? 37. Which of these doesn't belong to cat A. National Security and Civil Defence Corps family? B. Nigeria Stock and Credit Debit Company A. Lion C. Nigerian Security and Civil fence Corpse B. Leopard D. Nigeria Security and Civil Defence Corp C. Cougar D. Bear 47. What is the full meaning of IMF? A. Internal Memorandum fund 38. Which of these doesn't have MODE? B. International Monetary fund A. 20 40 20 30 20 C. Intervention Monetary fund B. 50 70 30 50 10 D. Intelligence Members forum C. 20 40 30 70 60 D. 20 20 20 60 10 48. What's the full meaning of NAFDAC? A. Nigerian Agency for food and drug 39. He is my ___. administration and control A. Coleague B. National Agency for food and drug B. Colleague administration and control C. Collige C. National Association of food and drug D. College accomplishment and control D. Nigeria Agency for food, drug, 40. Choose the correct spelling. administration and control A. Lieutenant B. Leftanant 49. What's the full meaning of NDLEA? C. Lietenant A. Nigerian Drugs Law Enforcement Agency D. lieutenat B. Nigeria Drug and Law Enforcement Association 41. The number of Geopolitical zones in C. National Drugs Law Enforcement Association Nigeria Is? D. National Drug Law Enforcement Agency A. Two B. Four 50. What modifies a verb? C. Six A. Preposition D. Eight www.prep.ngB. Noun C. Adverb 42. Which of these is a month with 30 days? A. D. Adjective November B. February 51. Where was FESTAC '77’ held? C. January A. Abuja D. December B. Kwara C. Benin 43. The first Nigerian to get a Nobel prize? D. Lagos A. Prof Chinua Achebe B. Dr Olusegun Obasanjo 52. Gombe state slogan is? C. Prof Wole Soyinka A. Jewel of the Savannah D. Dr Ngozi Okonjo-Iweala B. Pearl of Tourism C. The New World 44. What year was the first world Cup hosted? D. State of Harmony A. 1957 69

Download More FREE Past Questions at www.prep.ng

53. The First FIFA world cup hosted in Africa A. Has was in what year? B. Are A. 1960 C. Is B. 1998 D. Was C. 2010 D. 2018 62. The headquarter of AU is? A. Addis Ababa, Ethiopia 54. The same vowel sound is found in; Crude B. Casablanca, Morocco and? C. Abuja, Nigeria A. Stole D. Accra, Ghana B. Clock C. Rout 63. Full meaning of UTME? D. hoot A. Universal Tertiary Matriculation Examination B. Unified Technology Matriculation 55. There was____ much noise at night that Examination we couldn't sleep. C. University Tertiary Matriculation A. such a Examination B. that D. Unified Tertiary Matriculation Examination C. very D. so 64. A dark horse is? A. Someone who is dark in complexion 55. The title of a report should be ___. B. A workaholic of African descent A. unambiguous C. Someone who little is known about B. ambiguous D. An irate individual C. inciting D. verbose 65. The correct spelling is? A. Lolipop 56. A. Millennium B. Lollipop B. Millenium C. Lolippop C. Mileniuim D. Lollippop D. Milennium 66. She said she was disappointed _____ his 57. An intransitive verb is a ____. behaviour. A. verb with an object A. In B. verb with a subject B. For C. verb with no object C. At D. verb with only one object D. By

58. A. Demarcation 67. To hit someone below the belt ____is an B. Dermercation idiomatic expression connoting? C. Dermacation A. To fight with the individual D. Dermarcation B. To gain advantage over them www.prep.ngC. To defeat the individual 1 1 59. 125 ⁄3 × 49 ⁄2 × 100 D. To discipline them A. 45 B. 40 68. The wind blew and the leaves fell. This is C. 35 an example of a? D. 30 A. Compound Sentence B. Complex Sentence 60. Name of Current University of Ilorin C. Compound-complex sentence registrar? D. Simple Sentence A. Prof Abdulkareem Sulyman Age B. Mr Emmanuel Dada Obafemi 69. Whenever a goal is scored, fans jubilate. C. Dr Folaranmi Modupe Olowoleni The underlined is an example of what type of D. Prof Sylvia Omonirume Malomo sentence? A. Simple 61. Either the principal or the teachers B. Complex ____good. C. Compound 70

Download More FREE Past Questions at www.prep.ng

D. Compound-complex C. Ommision D. Ommission 70. The woman was anxious because her child was seriously ill in the hospital. The word 79. A. Chauffere synonymous to the underlined is? B. Chauffeur A. Eager C. Chaffeur B. Elated D. Chafeur C. Worried D. Interested 80. A. Corection B. Correcttion 71. Which state is referred to as pace setter? C. Correction A. Ebonyi D. Corecction B. Kebbi C. Jigawa 81. A. Disappear D. Oyo B. Disapear C. Disappere 72. _____is something both friends and family D. Disappaer look forward to with great joy. A. Retrenchment 82. A. Lesure B. Retirement B. Leisure C. Parsimony C. Liesure D. Divorce D. Lesiure

73. Having success gives one great ____. 83. A. Embarrass A. Fulfillment B. Ebarrass B. Fulfilment C. Embarass C. Fullfilment D. Embarras D. Fufilment 84. A. Etiqette 74. An organisation that controls climate B. Etiquette change in Nigeria is____? C. Etiquete A. NEMA D. Etiquetee B. CCRP C. FADAMA 85. A. Aceleration D. UN B. Accelleration C. Acceleration 75. A man bought a gross of oranges; A score D. Accelaretion was bad. What is the percentage of the good ones? 86. A. Acomodation A. 56% B. Accomondation B. 66% C. Acommodation C. 76% D. Accommodation D. 86% www.prep.ng 87. A. Weather 76. The president at the time Nigeria won her B. Waether first African Cup of Nation was? C. Wethaer A. Abdulsalam Abubakar D. Weater B. Olusegun Obasanjo C. Ibrahim Badamosi Banbagida 88. A. Collaegue D. Shehu Shagari B. Colleague C. Colleag 77. A. Referrendum D. Coleague B. Referedum C. Referendum 89. A. Kalaidoscope D. Renferedum B. Kaledoscope C. Kaleidoscope 78. A. Omission D. Kailedoscope B. Omision 71

Download More FREE Past Questions at www.prep.ng

90. A. Evironment A. 24 B. Enviroment B. 30 C. Eviromenr C. 36 D. Environment D. 32

91. A. Privileged 101. The head of a school is principal while the B. Priviledge head of a University is_____. C. Previledge A. chancellor D. Previlige B. vice Chancellor C. provost 92. A. Referred D. proprietor B. Refered C. Reffered 102. The millennium development Goal was D. Refferred introduced in _____. A. 2014 93. A. Dirrhae B. 2015 B. Diarrhoea C. 2016 C. Diarrhea D. 2018 D. Diarrhae 103. Nigeria became a republic in _____. 94. A. Flourescent A. 1st October, 2000 B. Fluorescent B. 1st October, 1960 C. Floweresent C. 1st October, 1959 D. Fluoriscent D. 1st October, 1963

95. Which of these has to do with Clusters? 104. Which of the following statements about a A. Monophthong rectangle is not true? B. Diphthong A. All sides are equal C. Vowels B. All angles are equal D. Consonant C. It has two diagonals D. It has four sides 96. I saw her____ through the door? A. Walk 105. Find the perimeter of a rectangle whose B. Walking length is 30cm and breadth is 0.15m. (answer C. Walks should be in cm) D. Walked A. 30.15cm B. 0.45 cm 97. Coyote is a member of the _____family. C. 90cm A. Cat D. 45cm B. Dog C. Bat 106. Mt. Everest is located in ____. D. Rodent A. London www.prep.ngB. Moscow 98. Circulation C. Beijing A. CIRculation D. Nepal B. cirCUlation C. circuLAtion 107. Abraham Lincoln was once a president of D. circulaTION _____. A. USA 99. He spoke to them in a language B. England ____shocking for word. C. Switzerland A. So D. France B. Very C. Much 108. The full meaning of EFCC is ___. D. Too A. Economic financial and crimes commission B. Economic financial crime commission 100. How many countries qualified for Russia C. Economic and Financial crime commission 2018 World Cup? D. Economic financial crimes commission 72

Download More FREE Past Questions at www.prep.ng

118. More than 30 million are infected with 109. Ade's friends gave a wide berth to the HIV/AIDS. boy that beat Ade. A. Compliment of the verb A. They attacked the boy B. Object of the verb B. They kept distance from the boy C. subject of the verb C. They abused the boy D. adverb of the verb D. They gave the boy a gift 119. "Beatrice is always chasing the 110. A trader made a 15% loss from the sale rainbow" means that ____. of a good. Find the ratio of selling price to cost A. Beatrice is always trying to get favour from price. superiors A. 3:20 B. Beatrice loves bright colours B. 20:8 C. Beatrice is materialistic C. 3:7 D. Beatrice is always after something she will D. 17:20 never get or attain

111. A. strenous 120. If (2,3,1) (3,6,4) = (3,3,1) and (3,0,3) B. strennuous (7,2,6) = (3,2,4). Find (2,2,1) (3,4,3) C. strennous A. (3,2,3) D. strenuous B. (3,0,1) C. (2,2,2) 112. A. Definately D. (2,2,1) B. Definitaly C. Definitely 121. /i:/ D. Definataly A. Kite B. eat 113. A. ocasion C. lit B. occassion D. right C. occasion D. ocassion 122. /g/ A. gnash 114. Superhuman ; 'super' here means___. B. king A. before C. rugged B. after D. wrong C. under D. beyond 123. The family is looking forward to celebrating ____. 115. The baby of an Elephant will never act A. Christmas festival like a buffalo. This illustration signify B. Christmas ceremony A. Self-knowledge C. Christmas B. Self-control D. Christmas holidays C. Self-Respect D. Pride www.prep.ng124. A. UNfortunately B. unFORtunately 116. What colour is often associated with hot C. unforTUnately temperatures? D. unfortuNATEly A. Yellow B. Blue 125. Is Sola with my CAR? C. Red A. Is Dupe with my car? D. Green B. Who is with my car? C. Is Sola with your car? 117. 35 + 2b−b2 D. Is Sola with my key? A. (35−b) (2+b) B. (35 + b)(1−b) 126. The information she gave me was C. (7+b)(5+b) invaluable. (synonyms) D. (7−b)(5+b) A. Necessary B. Useful C. Costless 73

Download More FREE Past Questions at www.prep.ng

D. Essential A. March 14 B. February 14 127. The girl's only job was to care ___her sick C. May 29 grandmother. D. March 29 A. For B. about 136. The man bought clothes, shoes and C. to mattresses in the supermarket. This sentence D. of contains how many clause(s)? A. One 128. The students as well as their principal B. Two ____in school last week. C. Three A. was D. Four B. is C. were 137. In letter writing, complimentary greetings D. are end with a/an ___. A. Full stop 129. The workers and not their manager B. Exclamation mark responsible for the crisis. C. Comma A. was D. Colon B. are C. is 138. Full stop is used in the ____of a D. were sentence. A. Beginning 130. It all depended on what he____. B. Middle A. wanted C. ending B. wants D. start C. will want D. has wanted 139. Kunle claimed he hit the girl inadvertently but we all know he did it. 131. ____you go, I will follow you. (Antonyms) A. whereever A. unintentionally B. werever B. forcefully C. wereever C. Deliberately D. wherever D. Obtrusively

132. Which currency in Nigeria was introduced 140. Let us say we agree. in 2005? A. Don't we A. ₦500 B. isn't it B. ₦200 C. Shall we C. ₦100 D. can't we D. ₦1000 141. In those days, things were easy. A young 133. Which of the followingwww.prep.ng punctuation marks man ____own a house. doesn't have a dot? A. could have A. hyphen B. might B. colon C. could C. semi colon D. may D. Exclamation mark 142. 25% of x is y, what is x? y 134. What category of states are in the same A. /4 geo-political zon? B. 4y A. Sokoto, Bornu, Kaduna, Zamfara C. y⁴ B. Kwara, Niger, Plateau, Yobe D. 4 C. Imo, Akwa-Ibom, Enugu, Bayelsa D. Kwara, Niger, Nasarawa, Plateau

135. The 2015 presidential election was held on ____. 74

Download More FREE Past Questions at www.prep.ng

SOLUTION TO 2018 POST UTME QUESTIONS

1. A 2. C 3. A 4. B 5. A 6. A 7. B 8. A 9. B

10. B 11. A 12. B 13. C 14. B 15. D 16. D

17. B 18. C 19. D 20. A 21. D 22. C 23. A

24. D 25. B 26. C 27. C 28. D 29. C 30. A

31. A 32. C 33. C 34. C 35. B 36. B 37. D

38. C 39. B 40. A 41. C 42. A 43. C 44. C

45. C 46. A 47. B 48. B 49. D 50. C 51. D

52. A 53. C 54. A 55. A 56. A 57. C 58. A

59. C 60. C 61. B 62. A 63. D 64. C 65. B

66. C 67. B 68. A 69. B 70. C 71. D 72. B

73. B 74. A 75. D 76. D 77. C 78. A 79. B

80. C 81. A 82. B 83. A 84. B 85. C 86. D

87. A 88. B 89. C 90. D 91. A 92. A 93. B

94. B 95. D 96. A 97. B 98. C 99. D 100. D

101. B 102. B 103. D 104. A 105. C 106. D

107. A 108. C 109. B 110. D 111. D 112. C

113. C 114. D 115. A 116. C 117. D 118. C

119. D 120. C 121. B 122. C 123. C 124. B

125. D 126. D 127. A 128. C 129. B 130. A

131. D 132. D 133. A 134. D 135. D 136. A www.prep.ng 137. C 138. C 140. C 141. C 142. B

75